MCQs 4dd Oral Pathology 2019

Reviewed by Editorial Team
The ProProfs editorial team is comprised of experienced subject matter experts. They've collectively created over 10,000 quizzes and lessons, serving over 100 million users. Our team includes in-house content moderators and subject matter experts, as well as a global network of rigorously trained contributors. All adhere to our comprehensive editorial guidelines, ensuring the delivery of high-quality content.
Learn about Our Editorial Process
| By Uhsmaster101
U
Uhsmaster101
Community Contributor
Quizzes Created: 1 | Total Attempts: 724
| Attempts: 724 | Questions: 200
Please wait...
Question 1 / 200
0 %
0/100
Score 0/100
1. Expansion of radicular cyst is due to:

Explanation

The expansion of a radicular cyst is caused by a combination of factors. Collagenases play a role in breaking down the connective tissue components, leading to the enlargement of the cyst. Continued epithelial proliferation also contributes to the expansion as the epithelial cells continue to multiply. Additionally, there is an increase in osmotic pressure in the cystic fluid, which further promotes the growth of the cyst. Therefore, all the options provided are true and contribute to the expansion of a radicular cyst.

Submit
Please wait...
About This Quiz
Oral Pathology Quizzes & Trivia

This quiz, titled 'MCQs 4DD Oral Pathology 2019', assesses knowledge in oral pathology with a focus on patient history taking, diagnosis, and management of dental conditions. It is... see moredesigned for dental professionals to enhance their understanding and approach to oral health care. see less

2. Factors that predispose to the development of denture stomatitis include:

Explanation

The correct answer is "All answers". This means that all of the factors listed (complete upper denture, poor dental hygiene, acrylic dentures, poorly fitting dentures, and old dentures) predispose to the development of denture stomatitis.

Submit
3. A 20 year old female student presents with red patches on her tongue that move from one location to another. They have been previously symptomless but have recently become sensitive to spicy foods and tongue has become sore. Student feels tired & week. The most likely cause of these symptoms are:

Explanation

The most likely cause of the symptoms described is iron deficiency anemia accompanying geographic tongue. Iron deficiency anemia can cause symptoms such as fatigue and weakness, and it can also affect the health of the tongue, leading to symptoms like red patches and sensitivity to spicy foods. The fact that the patches on the tongue are moving from one location to another is consistent with geographic tongue, which is a benign condition. Therefore, the correct answer is iron deficiency anemia accompanying geographic tongue.

Submit
4. The patient with a fissured tongue is advised to :

Explanation

Patients with a fissured tongue are advised to brush their tongue gently with a soft toothbrush and keep the fissures clean of debris and irritants. This is because debris and irritants can accumulate in the fissures, leading to discomfort and potential infection. Brushing the tongue helps to remove these particles and maintain oral hygiene.

Submit
5. All sialoliths affect the major salivary glands:

Explanation

The correct answer is 80%. This means that out of all sialoliths, 80% of them affect the major salivary glands. Sialoliths are calcified structures that can form in the salivary glands, causing blockages and leading to various symptoms such as pain and swelling. The fact that 80% of sialoliths affect the major salivary glands suggests that these glands are more commonly affected compared to the minor salivary glands.

Submit
6. Very common complication of dentigerous cyst is:

Explanation

Ameloblastoma is a very common complication of dentigerous cysts. Ameloblastoma is a benign tumor that originates from the cells responsible for forming tooth enamel. It can cause significant damage to the jawbone and surrounding tissues if left untreated. Dentigerous cysts, which are fluid-filled sacs that form around the crown of an unerupted tooth, can sometimes lead to the development of ameloblastoma. Therefore, it is important to monitor and treat dentigerous cysts to prevent the potential complication of ameloblastoma.

Submit
7. Most common etiology for radicular cyst is:

Explanation

The most common etiology for a radicular cyst is dental caries and trauma. Dental caries refers to tooth decay, which can lead to the formation of a cyst in the root of the tooth. Trauma, such as a physical injury to the tooth or surrounding tissues, can also cause a radicular cyst to develop. Both dental caries and trauma can disrupt the normal structure of the tooth and surrounding tissues, leading to the formation of a cyst.

Submit
8. Commonest site for sialolithiasis is:

Explanation

Sialolithiasis refers to the formation of salivary stones, which can obstruct the ducts and cause pain and swelling. The submandibular duct, also known as Wharton's duct, is the most common site for sialolithiasis. This is because the submandibular gland produces thicker saliva with a higher mineral content, making it more prone to stone formation. The stones can block the narrow duct, leading to the symptoms of sialolithiasis. The other options, such as minor salivary gland ducts, parotid ducts, and sublingual duct, are less commonly affected by sialolithiasis.

Submit
9. A biopsy would be of value in the diagnosis or oral lesion of:

Explanation

A biopsy would be of value in the diagnosis of an oral lesion for all the given conditions: amyloidosis, carcinoma in situ, tuberculosis, and lichen planus. Biopsy involves taking a small sample of tissue from the affected area for microscopic examination. This helps in determining the exact nature of the lesion and confirming the diagnosis.

Submit
10. Peripheral ossifying fibroma most commonly appears to originate from:

Explanation

Peripheral ossifying fibroma is a benign reactive lesion that commonly occurs in the gingiva. It typically arises from the interdental papilla, which is the triangular gum tissue located between adjacent teeth. This lesion is characterized by the formation of mineralized tissue, resembling bone or cementum. The interdental papilla is a common site for the development of peripheral ossifying fibroma due to its proximity to the teeth and the presence of local irritants such as plaque and calculus.

Submit
11. The tumor are soft to touch, movable, generally painless:

Explanation

Lipoma is the correct answer because it is a benign tumor made up of fat cells. Lipomas are typically soft to touch, movable, and generally painless. This is in contrast to other types of tumors like sarcoma, which are malignant and can be hard, fixed, and painful. Granulomas, on the other hand, are a type of inflammation and not tumors.

Submit
12. The main causes of the lipoma is :

Explanation

The main cause of lipoma is unknown. This means that the exact reason for the development of lipomas is not yet understood. While trauma, infection, and chronic irritation may contribute to the formation of lipomas in some cases, they are not considered as the main causes. Further research is needed to determine the underlying factors that lead to the development of lipomas.

Submit
13. Oral cytological smears are of no value in the diagnosis of:

Explanation

Oral cytological smears are not useful in diagnosing lipoma because lipomas are benign tumors composed of fat cells, which do not shed cells into the oral cavity. Cytological smears are more commonly used to diagnose lesions that involve epithelial cells, such as oral cancer or herpes infections.

Submit
14. The term of tumor which is a caused by Human herpesvirus 8 (HHV8), and it is associated herpesvirus, which one:

Explanation

Kaposi's sarcoma is a type of tumor that is caused by Human herpesvirus 8 (HHV8). It is an associated herpesvirus, meaning that the virus is closely linked to the development of this particular type of tumor. Lipoma, malignant, and benign are not specifically associated with HHV8 and do not have a direct link to this virus.

Submit
15. Elderly patients sometimes present with a fungal infection of the palate, beneath a denture, referred to as denture stomatitis. Which organism causes this condition?:

Explanation

Denture stomatitis is a fungal infection that occurs in elderly patients beneath a denture. The organism that causes this condition is Candida albicans. Candida albicans is a type of yeast that is commonly found in the oral cavity. It can overgrow and cause an infection in individuals with compromised immune systems or those who wear dentures. Treatment typically involves antifungal medications and proper denture hygiene.

Submit
16. Many diseases are due to infection with one type of micro-organism or another. Which one of the following options is the most likely example of an infection that is not caused by a micro-organism?:

Explanation

Creuzfeldt-Jakob disease is a rare, degenerative, and fatal brain disorder that is not caused by a micro-organism. It is caused by an abnormal protein called a prion. Prions are not considered micro-organisms as they lack genetic material and are not classified as living organisms. Therefore, Creuzfeldt-Jakob disease is the most likely example of an infection that is not caused by a micro-organism.

Submit
17. Which of the following options is the group of bacteria most likely to live in colonies arranged as chains of circular micro-organisms?:

Explanation

Streptococcus is the correct answer because it is a type of bacteria that is known to form chains or groups of circular microorganisms. This arrangement is characteristic of Streptococcus, making it the most likely option among the given choices. Bacilli, Lactobacilli, Spirochetes, and Staphylococcus do not typically form chains of circular microorganisms, which is why they are not the correct answer.

Submit
18. When body tissues become infected, they exhibit five classic signs of inflammation. Which is the last of these five signs to occur?:

Explanation

The last of the five classic signs of inflammation to occur is loss of function. This typically happens after the other signs, such as heat, pain, redness, and swelling, have already manifested. Loss of function refers to the impairment or inability of the affected body part or tissue to perform its normal activities or functions. It occurs as a result of the inflammatory response and can be temporary or permanent, depending on the severity of the infection and the extent of tissue damage.

Submit
19. Is a mild, but highly contagious viral infection common in young children, it occurs mainly in children under 10 years old but can also occur in adults.:

Explanation

Hand, foot, and mouth disease is a mild, but highly contagious viral infection that commonly affects young children. It is characterized by the presence of sores or blisters on the hands, feet, and inside the mouth. While it primarily occurs in children under 10 years old, it can also affect adults. The infection is typically spread through close contact with an infected person's saliva, blister fluid, or feces. Symptoms may include fever, sore throat, and a rash. Treatment usually involves managing symptoms and allowing the infection to run its course, as there is no specific cure.

Submit
20. Sign and symptom of Hand food moth diseas:

Explanation

The correct answer is "All answers." This is because all of the listed symptoms (fever, headache, sore throat, fatigue, irritability, loss of appetite, weight loss) can be signs and symptoms of Hand foot mouth disease.

Submit
21. Hand food mouth disease is the infection persons are most contagious during the:

Explanation

Hand, foot, and mouth disease is a highly contagious viral infection that primarily affects young children. The virus is most contagious during the first week of the illness. During this time, the infected person sheds the virus through respiratory secretions, saliva, and feces, making it easy for the virus to spread to others. It is important to take necessary precautions to prevent the spread of the disease, especially during the first week when the risk of transmission is highest.

Submit
22. Signe and symptom of the Halitosis:

Explanation

The given correct answer is "All answers." This means that all of the options listed (a white or yellow coating on your tongue, people seem to avoid direct contact with you and step back from you when you are talking to them, and a smelly odor when you open your mouth) are signs and symptoms of Halitosis.

Submit
23. Management of the Halitosis:

Explanation

The correct answer is "All answers." This means that all of the options listed (patient education, avoiding smoking and food, decreasing alcohol and coffee intake, and using oral antiseptics) are effective in managing halitosis. Each of these approaches can help improve oral hygiene and reduce bad breath.

Submit
24. The following are the factors known to cause Apthouse Stomatitis:

Explanation

The correct answer is "All answers." This means that all of the factors listed (trauma, immunologic abnormalities, stress and emotional upset, and allergic factors) are known to cause Apthouse Stomatitis. This suggests that Apthouse Stomatitis can be caused by a combination of these factors, rather than just one specific cause.

Submit
25. Treatment management of the Apthouse stomatitis:

Explanation

The correct answer is "All answers." This means that all of the options listed (diet supplement, tropical corticosteroid, and tetracycline mouthwashes) are appropriate for the treatment management of Apthouse stomatitis. This suggests that a combination of these treatments may be necessary to effectively manage the condition.

Submit
26. You are dentist in general practice. A patient attends your practice complain of a dry mouth or xerostomia. the cause of dry mouth:

Explanation

The correct answer is "All answers" because dry mouth or xerostomia can be caused by a variety of factors including side effects of certain medications, certain diseases and infections, nerve damage, dehydration, lifestyle choices, Sjogren's syndrome, infection, and cancer treatments.

Submit
27. An inflammation or swelling of the tissue lining the sinus, this is the:

Explanation

Sinusitis is the correct answer because it refers to the inflammation or swelling of the tissue lining the sinus. Rhinitis refers to inflammation of the nasal lining, not specifically the sinus lining. Granuloma refers to a type of tissue growth or lesion, which is not necessarily related to sinus inflammation. Allergies can cause symptoms similar to sinusitis, but they are not the same condition. Therefore, sinusitis is the most appropriate answer.

Submit
28. The maxillary Sinus :

Explanation

The maxillary sinus is innervated by branches of the maxillary division of the trigeminal nerve. This means that the sensory information from the maxillary sinus is carried by the trigeminal nerve, which is responsible for sensation in the face. The other statements in the question are incorrect or unrelated to the innervation of the maxillary sinus.

Submit
29. A patient is most likely to experience pain due to infection of the ethmoidal air cells sinus:

Explanation

A patient is most likely to experience pain due to infection of the ethmoidal air cells sinus between the eyes. The ethmoidal air cells are located between the eyes and inflammation or infection in this area can cause pain and discomfort in that specific region.

Submit
30. Mary 26- year-old woman is worried and embarrassed because of her mouth bad breath, the assessing of prevention and management this problem:

Explanation

The correct answer is "All answers" because all of the given suggestions can help in preventing and managing bad breath. Treating any medical causes, avoiding mal-odor foods, avoiding habits that worsen breath odor, brushing teeth after eating, keeping oral hygiene regular, and brushing the tongue before going to bed are all effective strategies to address the issue of bad breath.

Submit
31. The patient 30 year old came to the dental clinic, Dentist asking her about the communicate the medical and dental history for leading to the examination and diagnosis for management in her condition, which one the following is likely not meaning of the history taking :

Explanation

The explanation for the correct answer is that asking the patients to tell the Mather for the first visit in a hospital is not a meaning of history taking in dental practice. History taking in dental practice refers to the process of gathering information about the patient's medical and dental history in order to understand their condition and provide appropriate management. It involves discussing the patient's concerns, using methodical questioning to gather further details, and establishing a baseline for examination and diagnosis. Asking about the patient's first visit to a hospital is not directly related to dental history and does not contribute to the understanding of the patient's dental condition.

Submit
32. Mary 36 year old came to the dental clinic, he Complain of Chronic gum disease, and dental history were finding normal and have the experience of bleeding gum for a year, Which the following the most likely to interview is the first personal information in the history taking focus?:

Explanation

The most likely first personal information to focus on in the history taking interview would be Name, age, sex. This is because the patient's name is necessary for identification purposes, while their age and sex can provide important context for understanding their overall health and potential risk factors for gum disease. The other options such as address, marital status, weight, height, phone, e-mail, and handy number may be relevant for administrative purposes or further communication, but they are not as crucial in addressing the patient's immediate dental concern.

Submit
33. The Patient 16 year old, he is the extremely pain the tooth 36 in the might night, the past dental history is the history of dental treatment undergone by the patient, along with patients experience before, during and after the dental treatment, likely one the following the Dentist receive from patients? :

Explanation

The dentist is likely to receive information about the patient's dental treatment history, including their experience before, during, and after the treatment, during their last visit.

Submit
34. Which one of the following is least likely indication for antibiotic prophylaxis to prevent bacterial endocarditis:

Explanation

Diabetes mellitus is least likely to be an indication for antibiotic prophylaxis to prevent bacterial endocarditis. Antibiotic prophylaxis is typically recommended for individuals with congenital heart disease, previous history of endocarditis, rheumatic fever, and valvular heart disease, as they have an increased risk of developing bacterial endocarditis. However, diabetes mellitus does not directly increase the risk of bacterial endocarditis, and therefore, antibiotic prophylaxis is not necessary for individuals with this condition.

Submit
35. Scarlet fever is a complication of a streptococcal oropharyngeal infection. It occurs cause by:

Explanation

Scarlet fever is a complication of a streptococcal oropharyngeal infection, and the correct answer is β-hemolytic streptococci. This type of streptococcus is known to produce toxins that cause the characteristic symptoms of scarlet fever, such as a red rash, high fever, and sore throat. Staphylococcus aureus, α-hemolytic streptococcus, and δ-hemolytic streptococcus are not typically associated with scarlet fever.

Submit
36. Most common route of entry mycobacterium into the body is via:

Explanation

The most common route of entry for mycobacterium into the body is through the respiratory tract. This is because mycobacterium, including the bacteria that cause tuberculosis, are primarily transmitted through inhalation of respiratory droplets containing the bacteria. When an infected individual coughs or sneezes, these droplets can be inhaled by others, allowing the bacteria to enter the respiratory system and potentially cause infection.

Submit
37. Commonest candidial species causing oral candidiasis are:

Explanation

Candida albicans is the most common species causing oral candidiasis. This is because it is a natural inhabitant of the oral cavity and can overgrow when there is an imbalance in the oral microbiota or when the immune system is weakened. Candida tropicalis, Candida krusei, and Candida stellatoidea are also known to cause oral candidiasis, but they are less common compared to Candida albicans.

Submit
38. The patient 30 year old came to the dental clinic, Dentist asking her about the communicate the medical and dental history for leading to the examination and diagnosis for management in her condition, which one the following is likely not meaning of the history taking?

Explanation

History taking in a dental clinic refers to the process of gathering information about the patient's medical and dental history in order to conduct an examination and make a diagnosis. It involves discussing with the patient about their concerns and using methodical questioning to gather further details. Asking the patient to tell their mother about their first visit to a hospital is not a part of history taking in this context.

Submit
39. Mary 36 year old came to the dental clinic, he Complain of Chronic gum disease, and dental history were finding normal and have the experience of bleeding gum for a year, Which the following the most likely to interview is the first personal information in the history taking focus?

Explanation

The most likely first personal information to focus on in the history taking is the name, age, and sex. This information is important for identifying the patient and determining their demographic characteristics. It also helps in keeping track of the patient's medical records and history. The other options such as address, marital status, weight, height, phone, email, and handy number may be relevant for certain aspects of the patient's history, but they are not as crucial as the basic identification information.

Submit
40. Mary 36 year old came to the dental clinic, he Complain the corner of the Lip is the inflammation, the dental history were finding normal, Which the following referred to as:

Explanation

Cheilitis refers to the inflammation of the corners of the lips. In this case, Mary, a 36-year-old patient, presented to the dental clinic with a complaint of inflammation in the corner of the lip. The dental history was found to be normal. Therefore, the correct answer is Cheilitis.

Submit
41. The Patient 16 year old, he is the extremely pain the tooth 36 in the might night, the past dental history is the history of dental treatment undergone by the patient, along with patients experience before, during and after the dental treatment, likely one the following the Dentist receive from patients?

Explanation

The correct answer is "Last visit". This is because the question is asking about the likely information that the dentist would receive from the patient. In this case, the patient is experiencing extreme pain in tooth 36 at night and the question mentions the patient's past dental history. The most relevant information about the patient's dental history would likely come from their last visit to the dentist, as it would provide the most recent information about any treatments or experiences related to the tooth in question.

Submit
42. Marya, she is 45 year old, she come the dental clinic because she is problem about the cellulitis from the tooth 28 but she have the allergic, diabetic, and her mother to die by cancer of the breast after the dentist ask her and take noted history in the observation. Which one following the mostly the genetic condition we are to question ask the patient seen it in the history of:?

Explanation

The correct answer is "Family" because the question is asking about a genetic condition that the patient may have seen in their family history.

Submit
43. Smoking, alcohol, Drug and accommodation is the:?

Explanation

The correct answer is social history because smoking, alcohol, drug use, and accommodation are all factors that are related to an individual's social habits and lifestyle choices. Medical history would focus more on past and current medical conditions and treatments, past dental history would focus on previous dental treatments and oral health, and family history would focus on the medical conditions and health history of the individual's family members. Therefore, social history is the most appropriate category for these factors.

Submit
44. The excessive formation of scar tissue is called:

Explanation

Keloid is the correct answer because it refers to the excessive formation of scar tissue. Keloids are raised, thickened areas of scar tissue that extend beyond the boundaries of the original wound. They can be caused by an overproduction of collagen during the healing process and are more common in individuals with darker skin tones. Fibroma, myxoma, myoma, and carcinoma are not specifically related to the excessive formation of scar tissue.

Submit
45. The classical method of examination follows the following sequence: ?

Explanation

The correct answer is "All answers" because the classical method of examination involves performing all of the listed actions in sequence. This means that before moving on to the next step, the examiner must first obtain all of the necessary answers from the patient. Only after gathering this information can they proceed with the physical examination, which includes inspection, auscultation, palpation, and percussion.

Submit
46. A biopsy would be of value in the diagnosis or oral lesion:

Explanation

A biopsy would be of value in the diagnosis of an oral lesion because it allows for the examination of tissue samples under a microscope to determine the presence of abnormal cells or other pathological changes. In the case of amyloidosis, carcinoma in situ, tuberculosis, and lichen planus, a biopsy can help confirm the diagnosis by identifying specific characteristics or markers associated with each condition. Therefore, all the options provided are true, indicating that a biopsy would be beneficial in diagnosing any of these oral lesions.

Submit
47. Normal per microliter of the Platelets from:?

Explanation

The normal range for the number of platelets per microliter of blood is 150,000 to 300,000. This range is considered normal because it represents the typical amount of platelets found in a healthy individual's blood. Platelets are important for blood clotting and maintaining overall hemostasis in the body. Any values below or above this range may indicate a potential health issue, such as a bleeding disorder or a bone marrow disorder.

Submit
48. The function of the platelets:?Primary hemostasis : platelet plug Seconded hemostasis : the coagulation cascade

Explanation

The correct answer is "All answers." The function of platelets includes primary hemostasis, where they form a platelet plug to stop bleeding, and secondary hemostasis, where they participate in the coagulation cascade. Platelets also play a role in vasculature maintenance and act as a reservoir for soluble factors. Therefore, all of the given answers are correct.

Submit
49. Often called good cholesterol ; because it remove excess cholesterol and caries it to the liver for remove, is the:?

Explanation

High-density lipoprotein (HDL) is often called good cholesterol because it removes excess cholesterol from the bloodstream and carries it to the liver for removal. HDL helps to prevent the buildup of plaque in the arteries, reducing the risk of heart disease. LDL, VLDL, triglycerides, and cholesterol are not typically referred to as good cholesterol and do not have the same beneficial effects on heart health as HDL.

Submit
50. The radiography, pule vitality, Blood test, Biopsy and study model is the:?

Explanation

The correct answer is "Para-clinic examination." Para-clinic examination refers to diagnostic procedures that are performed outside of the clinical setting, such as radiography, pulse vitality, blood tests, biopsy, and study models. These tests are used to gather additional information about a patient's condition and aid in the diagnosis and treatment planning process. Extra oral and intra oral examinations, on the other hand, refer to clinical examinations that are conducted within the oral cavity or on the outside of the oral cavity, respectively. Therefore, the correct answer is para-clinic examination as it encompasses all the mentioned diagnostic procedures.

Submit
51. The specific reason for which the patient has presented himself to the clinic:?

Explanation

The correct answer is "The chief complaint." The chief complaint refers to the specific reason for which the patient has presented themselves to the clinic. It is the main issue or symptom that the patient is experiencing and wants to address during their visit.

Submit
52. Cold test is important for dental test, is mean that the:?

Explanation

The explanation for the correct answer, "Thermal test," is that a cold test is a type of thermal test that is important for dental testing. This test involves the application of a cold stimulus to a tooth to assess its sensitivity and determine if there are any underlying dental issues. Therefore, the correct answer is thermal test because it accurately represents the purpose and significance of a cold test in dental testing.

Submit
53. We know about inflammation apex, we know about around tooth is the:?

Explanation

The correct answer is Percussion test. The question is asking about the method used to determine the inflammation apex or the area around the tooth. Percussion test is a diagnostic technique where a gentle tapping or striking of the tooth is done to assess any pain or discomfort, which can indicate inflammation or infection around the tooth. This test helps in identifying the source and extent of the inflammation.

Submit
54. Medical test is the:?

Explanation

The correct answer is "All answers" because a medical test typically involves measuring various vital signs, such as temperature, pulse, blood pressure, and respiration. These measurements provide important information about a person's health and can help diagnose and monitor medical conditions. Therefore, all of these answers are correct as they are all part of a comprehensive medical test.

Submit
55. An enamel defect resulting from the incomplete formation of the enamel matrix is called:

Explanation

Enamel hypoplasia is the correct answer because it refers to a condition where the enamel matrix is not fully formed, resulting in a defect in the enamel. This defect can cause the enamel to be thin, weak, or have pits and grooves. Enamel pearl, hypercementosis, and dentin dysplasia are not related to the incomplete formation of the enamel matrix, making them incorrect answers.

Submit
56. The term of the treatment refer to the?

Explanation

The term of the treatment refers to the management and care of a patient or the combating of disease. This includes all the actions and interventions taken to improve the patient's condition and alleviate symptoms, as well as the strategies used to fight against the disease itself. It encompasses various approaches such as medication, therapy, surgery, and other medical interventions aimed at providing the best possible outcome for the patient.

Submit
57. Head and neck cancer treatment is therapy using the:

Explanation

Head and neck cancer treatment typically involves the use of radiotherapy. Radiotherapy uses high-energy radiation to target and kill cancer cells in the affected area. It is a common treatment option for head and neck cancer as it can effectively shrink tumors and destroy cancer cells while minimizing damage to surrounding healthy tissues. This therapy can be delivered externally (external beam radiotherapy) or internally (brachytherapy), depending on the specific case. Drug and surgery may also be used in combination with radiotherapy, but radiotherapy is the primary treatment modality for head and neck cancer.

Submit
58. Tooth removable, Gum open flap, Bone graft, Oral cancer: take out tumor, it is the type treatment of the?:

Explanation

The correct answer is Surgical. The given treatment options, such as removing a tumor, tooth removable, gum open flap, and bone graft, all involve surgical procedures. Surgical treatment is often necessary for oral cancer and other dental and gum-related conditions that require physical intervention and removal of affected tissues. Pharmacological treatment involves the use of medications, radiotherapy uses radiation to treat cancer, and prevention focuses on measures to prevent the development of oral health issues.

Submit
59. Which of the following statement are true regarding “hemengioma”:

Explanation

Hemangiomas are abnormal growths of blood vessels that can range in size from small to very large. They can appear at birth or shortly thereafter, or they can develop later in life. Hemangiomas can enlarge to alarming sizes and in certain locations, they can interfere with the proper development and functions of organs. Therefore, all of the given statements are true regarding hemangiomas.

Submit
60. Type lichen Planus is? :

Explanation

The correct answer is "Reticular and plaque." Lichen Planus is a chronic inflammatory condition that affects the skin and mucous membranes. It is characterized by the presence of reticular (lace-like) and plaque-like lesions on the affected areas. These lesions can be red, itchy, and may have a shiny or scaly appearance. Other characteristics of lichen planus include Wickham striae (white lines or dots) and hyperkeratosis (thickening of the outer layer of the skin).

Submit
61. Wickham’s strie are seen in:

Explanation

Wickham's striae are a characteristic feature of lichen planus. Lichen planus is an inflammatory skin condition that affects the skin, mucous membranes, nails, and hair. Wickham's striae are fine, white lines or dots that appear on the surface of the affected skin or mucous membranes. These striae are caused by hyperkeratosis, which is the thickening of the outer layer of the skin. Therefore, the presence of Wickham's striae is a diagnostic feature of lichen planus.

Submit
62. The term xerostomia denotes:

Explanation

Xerostomia refers to the condition of having a dry mouth. This can be caused by a decrease in saliva production or an alteration in the composition of saliva. It can lead to discomfort, difficulty in speaking and swallowing, and an increased risk of dental problems. Dryness of the eye, dryness of the stomach, and increased salivation are not the correct definitions for xerostomia.

Submit
63. All of the following may lead to xerostomia except:

Explanation

Xerostomia, also known as dry mouth, is a condition characterized by a lack of saliva production. Aging, glandular aplasia, and severe vomiting can all contribute to xerostomia as they can affect the functioning of salivary glands. However, vitamin C deficiency is not typically associated with xerostomia. Vitamin C deficiency, also known as scurvy, primarily affects the connective tissues in the body and can lead to symptoms such as fatigue, bleeding gums, and joint pain, but it does not directly cause xerostomia.

Submit
64. During oral examination of 57 year old man a large white keratotic patch that covers the entire palate is noted. Some “red dots” are also seen in this patch. The most probable explanation is that the patient is a:

Explanation

The presence of a large white keratotic patch that covers the entire palate, along with the presence of "red dots" within the patch, suggests a condition called leukoplakia. Leukoplakia is commonly associated with tobacco use, particularly pipe smoking. Therefore, the most probable explanation for the patient's condition is that he is a pipe smoker.

Submit
65. A malnourished child shows ulceration of gingival and punched out lesion, necrotic gingival margin, which one the Diagnosis is:

Explanation

The correct answer is ANUG (Acute Necrotizing Ulcerative Gingivitis). ANUG is a condition characterized by ulceration of the gingiva, punched out lesions, and necrotic gingival margins. It is commonly seen in malnourished individuals and is caused by bacterial infection, particularly with a combination of anaerobic bacteria. Herpetic stomatitis is caused by the herpes simplex virus and presents with small, painful ulcers. NOMA is a severe gangrenous infection that affects the face and mouth, typically seen in malnourished children. Apthous ulceration refers to recurrent painful ulcers in the mouth, but it is not associated with the specific findings mentioned in the question.

Submit
66. What are the shape of the Aphthous ulcers?:

Explanation

Aphthous ulcers are commonly round or oval in shape. This can be observed through their appearance and the way they develop on the mucous membranes of the mouth. The round or oval shape distinguishes them from other types of ulcers, such as triangular or irregular-shaped ulcers.

Submit
67. Sing and symptoms of the Dry socket is following excepts one:

Explanation

The given answer is "Headache, Malaise, Lymph nodes ulcer". This is because the symptoms of dry socket do not include headache, malaise, or lymph node ulcers. The other options listed, such as an empty-looking socket, visible bone in the socket, pain increase, pain radiating, bad breath, unpleasant taste, and lymph nodes, are all associated with dry socket.

Submit
68. The lesion size is greater than one centimeter in diameter, They tend to be 10 mm or larger across, Usually only one or two appear at a time. It is:

Explanation

The correct answer is The Major aphthous ulceration. The explanation is that major aphthous ulcers are characterized by larger lesion sizes, usually greater than one centimeter in diameter. They tend to appear as one or two ulcers at a time. This is in contrast to minor aphthous ulcers, which are smaller in size and can occur in greater numbers. Erythema multiform ulcers and herpetiform aphthous ulcers are different conditions with their own distinct characteristics.

Submit
69. Common pathological finding of labial mucosa:

Explanation

The labial mucosa can present with various pathological findings. Apthous ulcers are painful sores that can occur on the mucosa. Mucoceles are cysts that form when saliva becomes trapped in a minor salivary gland. Fibrosing mucocels are mucoceles that have become chronically inflamed and fibrotic. Therefore, all of the given answers (Apthous, Mucocele, and Fibrosing mucocel) are common pathological findings of the labial mucosa.

Submit
70. The cause of the Benign migratory glossitis is the:

Explanation

The correct answer is "All answers." Benign migratory glossitis, also known as geographic tongue, is a condition characterized by irregular, smooth, red patches on the tongue that can change in shape and location over time. The exact cause of this condition is unknown, but it is believed to be multifactorial. Stress, allergies, irritation, vitamin deficiencies, fissured tongue, atopic deficiencies, and yeast infections have all been suggested as possible contributing factors to the development of benign migratory glossitis. Therefore, it is likely that all of the listed answers can play a role in causing this condition.

Submit
71. Candidiasis does occur under conditions such as :

Explanation

The correct answer is "All answers" because candidiasis, a fungal infection caused by the Candida fungus, can occur under various conditions such as antibiotic therapy, diabetes, xerostomia (dry mouth), and weakened immunologic reactions. These conditions can weaken the immune system, disrupt the natural balance of microorganisms in the body, and create an environment that is favorable for the growth of Candida fungus, leading to candidiasis.

Submit
72. White patches (Plaques) on the tongue, inner cheeks, sometimes on the roof of the mouth that can be wipe of, leaving behind red areas that may bleeding, Redness inside mouth and throat, Cracks at the corners of the mouth, Painful, burning sensation in the mouth, loss of taste, there are the Symptom of:

Explanation

The given symptoms, such as white patches on the tongue and inner cheeks that can be wiped off, redness inside the mouth and throat, and painful burning sensation, are characteristic of oral thrush candidiasis. This condition is caused by an overgrowth of the Candida fungus in the mouth. Ulceration and apthous stomatitis may also cause similar symptoms, but the presence of white patches that can be wiped off suggests oral thrush candidiasis as the correct answer. Inhaled corticosteroid medication for asthma is not directly related to these symptoms.

Submit
73. The Symptom of Oral thrush candidiasis like:

Explanation

The correct answer is "All answers" because oral thrush candidiasis can present with all of the mentioned symptoms. White patches on the tongue, inner cheeks, and sometimes on the roof of the mouth are common signs of oral thrush. Cracks at the corners of the mouth can also be a symptom. Additionally, individuals with oral thrush may experience a painful, burning sensation in the mouth and a loss of taste. Therefore, all of the given symptoms can be indicative of oral thrush candidiasis.

Submit
74. Oral Mucosa and soft tissue is the:?

Explanation

The correct answer is "All answers." The oral mucosa and soft tissue include the lips, buccal and vestibular mucosa, gingiva, hard and soft palates, uvula, pharynx, tongue, floor of the mouth, tonsils, salivary glands, frenum, and lingual frenum.

Submit
75. These elongated papillae become stained by food and tobacco, producing the name:

Explanation

Black hairy tongue is the correct answer because it is a condition where the papillae on the surface of the tongue become elongated and stained, giving the tongue a black or dark appearance. This staining can be caused by food and tobacco. Geographic tongue refers to a condition where the tongue has patches that resemble a map, while fissure tongue is characterized by grooves or clefts on the surface of the tongue. Erythema migrans is a skin rash associated with Lyme disease and is unrelated to the appearance of the tongue.

Submit
76. The most effective anti-plaque agent is:

Explanation

Chlorhexidine is considered the most effective anti-plaque agent because it has strong antimicrobial properties and can effectively kill bacteria that cause plaque formation. It is commonly used in mouthwashes and oral rinses to control plaque and prevent gum diseases. Chlorhexidine has a long-lasting effect and can inhibit the growth of bacteria for up to 12 hours. It is also effective against a wide range of bacteria, making it a popular choice for oral hygiene products.

Submit
77. Denture stomatitis is a clinical diagnosis based on the :

Explanation

Denture stomatitis is diagnosed based on the pattern of redness and swelling that follows the shape of the area where the denture contacts the mouth. This means that the redness and swelling will be localized to the specific areas where the denture comes into contact with the gums. Microbiological swabs can be taken from the palate and/or denture surface to confirm the presence of yeasts, which are often associated with denture stomatitis. Additionally, examining the denture itself may reveal some of the factors that contribute to the development of denture stomatitis.

Submit
78. All of the following are etiologically associated with aphthous ulcers except:

Explanation

Aphthous ulcers, also known as canker sores, are painful sores that form on the inside of the mouth. They are typically caused by a combination of factors, including trauma, stress, smoking cessation, microbial agents, and genetic factors. However, diabetes mellitus is not directly associated with the development of aphthous ulcers. Therefore, it is the exception among the listed etiological factors.

Submit
79. The most common location of dentigerous cysts are except one:

Explanation

The most common location for dentigerous cysts is usually the mandibular third molars, followed by the maxillary canines. However, the maxillary third molars are not a common location for dentigerous cysts.

Submit
80. . For severe cases of RAS and ulcerations associated with Behcet’s disease and HIV that have failed to respond to topical & systemic steroids, the drug of choice for patients excluding pregnant mothers is:

Explanation

Thalidomide is the correct answer because it has been shown to be effective in treating severe cases of RAS (Recurrent Aphthous Stomatitis) and ulcerations associated with Behcet's disease and HIV that have not responded to topical and systemic steroids. Thalidomide has immunomodulatory and anti-inflammatory properties that can help reduce inflammation and promote healing in these conditions. It is important to note that thalidomide should not be used in pregnant mothers due to its teratogenic effects.

Submit
81. There are factors to lead to the Sialolithiasis:

Explanation

Sialolithiasis is caused by various factors, and all the given options contribute to its development. Dehydration leads to the thickening of saliva, making it more prone to forming stones. Not eating enough reduces the demand for saliva production, which can also contribute to the formation of sialoliths. Additionally, abnormalities in calcium metabolism can lead to the deposition of calcium salts in the salivary glands, further increasing the risk of sialolithiasis. Therefore, all the provided answers are correct in explaining the factors that can lead to sialolithiasis.

Submit
82. The most frequent cause of xerostomia is:

Explanation

Xerostomia refers to the condition of having a dry mouth. The most frequent cause of xerostomia is drugs. Various medications, such as antihistamines, decongestants, antidepressants, and diuretics, can cause a decrease in saliva production, leading to dry mouth. This can be a side effect of these drugs and can be temporary or long-term depending on the medication being used.

Submit
83. For half the patient with erythema multiform the precise cause is:

Explanation

The precise cause of erythema multiform is unknown for half of the patients. This means that researchers and medical professionals have not been able to identify a specific virus, candida, or bacteria that is responsible for causing this condition in these individuals. The lack of knowledge about the exact cause can make it challenging to develop targeted treatments for erythema multiform.

Submit
84. Most common benign soft –tissue tumor of oral cavity is:

Explanation

A fibroma is the most common benign soft-tissue tumor of the oral cavity. It is a benign growth that arises from the connective tissue, usually due to irritation or trauma. Fibromas are typically firm, smooth, and painless, and they can occur anywhere in the oral cavity. Lipomas are also benign tumors, but they are more commonly found in other areas of the body rather than the oral cavity. Papillomas are caused by the human papillomavirus (HPV) and are usually found on the skin or mucous membranes, but not specifically in the oral cavity. Chondromas, on the other hand, are tumors that arise from cartilage and are extremely rare in the oral cavity.

Submit
85. Diagnostic of lipoma depend on the:

Explanation

The correct answer is "All answers". This means that the diagnostic of lipoma depends on all the options provided: physician examination, laboratory data review and biopsy, as well as MRI and CT scan. All of these methods are commonly used in diagnosing lipoma and are necessary to accurately assess and confirm the presence of lipoma in a patient.

Submit
86. A patient presents with asymptomatic, smooth, circumscribed red area in the midline anterior to circumvallate papillae on the dorsum of tongue with microscopic evidence of epithelial hyperplasia. Most probable diagnostic of the condition is:

Explanation

The patient's presentation of an asymptomatic, smooth, circumscribed red area in the midline anterior to circumvallate papillae on the dorsum of the tongue is consistent with median rhomboid glossitis. This condition is characterized by a loss of filiform papillae in the midline of the tongue, resulting in a smooth, red area. The microscopic evidence of epithelial hyperplasia further supports this diagnosis. Geographic tongue, hairy tongue, and lingual thyroid do not typically present with these specific features.

Submit
87. The “ Ghon complex” is associate with:

Explanation

The "Ghon complex" refers to a specific radiographic finding in the lungs that is associated with primary childhood tuberculosis. This complex consists of a small, calcified focus of infection (Ghon focus) along with an associated lymph node enlargement (Ghon lymph node). It is a characteristic feature of primary tuberculosis in children and is often seen in the lower part of the lung. The other options, adrenogenital syndrome, histiocytosis, and AIDS, are not specifically associated with the Ghon complex.

Submit
88. There are many different types of bacteria normally present in the oral cavity. Which of the following is the name of the one most likely to be associated with the onset of dental caries?:

Explanation

Streptococcus is the most likely bacteria to be associated with the onset of dental caries. Streptococcus mutans, in particular, is known to produce acid as a byproduct of sugar metabolism, which can lead to the demineralization of tooth enamel and the development of dental caries. This bacterium is commonly found in the oral cavity and is known for its ability to adhere to tooth surfaces, forming dental plaque. Therefore, its presence and activity contribute significantly to the development of dental caries.

Submit
89. A patient attends the surgery with bilateral swelling of the parotid salivary glands. Which condition are they suffering from?:

Explanation

The patient is suffering from mumps, which is characterized by bilateral swelling of the parotid salivary glands. Mumps is a viral infection that primarily affects the salivary glands and is spread through respiratory droplets. It commonly occurs in children and can cause symptoms such as fever, headache, and muscle aches in addition to the swelling of the salivary glands. Other options such as diphtheria, measles, poliomyelitis, and rubella do not typically present with bilateral swelling of the parotid salivary glands.

Submit
90. Hand food mouth disease is cause by :

Explanation

Hand, foot, and mouth disease is caused by Enterovirus71 (EV 71). This virus is a member of the Enterovirus genus, specifically the species Enterovirus A. It is highly contagious and primarily affects infants and young children. The virus is transmitted through close contact with respiratory secretions, saliva, feces, and fluid from the blisters or sores of infected individuals. Symptoms of hand, foot, and mouth disease include fever, sore throat, rash, and blisters on the hands, feet, and mouth. While other enteroviruses such as EV 72 and EV 73 exist, they are not associated with hand, foot, and mouth disease. Coxsackieviruses, another group of enteroviruses, can also cause similar symptoms but are not the primary cause of hand, foot, and mouth disease.

Submit
91. How many days of the Hand food moth disease, after fever onset, painful sore usually develop in the mouth. They begin as small red spots that blister and then often become ulcers

Explanation

After the onset of fever in Hand food moth disease, painful sores usually develop in the mouth within one or two days. These sores start as small red spots, which then blister and often turn into ulcers.

Submit
92. The main cause of the halitosis is the:

Explanation

The correct answer is "All answers" because neglecting tooth brushing and flossing can lead to the buildup of plaque and bacteria in the mouth, which can cause bad breath. Gingivitis, especially necrotizing gingivitis, is an inflammation of the gums that can also contribute to halitosis. Dental decay, such as cavities, can result in the release of foul-smelling gases that cause bad breath. Therefore, all of these factors can contribute to halitosis.

Submit
93. Sinusitis is inflammation of the sinuses that results in symptoms such as:

Explanation

The correct answer is "All answers" because sinusitis can cause all of the symptoms listed, including thickened nasal drainage, nasal congestion, facial pain or pressure, halitosis, dental pain, fever, fatigue, and cough. Sinusitis is characterized by inflammation of the sinuses, which can lead to these various symptoms.

Submit
94. Which of the following paranasal sinuses open into the middle meatus?:

Explanation

The maxillary sinuses open into the middle meatus.

Submit
95. Burning Mouth Syndrome affects mostly on :

Explanation

Burning Mouth Syndrome primarily affects women. This could be due to hormonal changes, as women experience more hormonal fluctuations throughout their lifetime compared to men. Additionally, women tend to have a higher prevalence of certain risk factors associated with the condition, such as nutritional deficiencies, psychological factors, and autoimmune disorders. While men and other age groups can also be affected by Burning Mouth Syndrome, it is more commonly observed in women.

Submit
96. The patient 37 year old came to the dental clinic, he Complain of Chronic halitosis disease, and medical history were finding normal beside gastritis and have the experience of influct gastric for a year, Which the following the most likely to interview is a method of obtaining the history?:

Explanation

The most likely method of obtaining the history in this scenario would be to interview the patient. Since the patient is the one experiencing the symptoms and has the medical history, they would be the best source of information for the practitioner to gather relevant details about their condition. The learner, practitioner, and instructor may play different roles in the dental clinic, but in this case, it is the patient who can provide the necessary information for the history-taking process.

Submit
97. 25 year old of the women to come the hospital every year for the medical and dental check up, the list of the Medical history is the information about certain medical conditions for the dental management, which one most likely in the condition my affective during to the:

Explanation

The most likely condition that may affect the dental practices is the medical complications. This is because the medical history provides information about certain medical conditions that may have an impact on the dental management and treatment provided. These medical complications can affect the type of treatment that can be given and may require special considerations or precautions during dental care.

Submit
98. The patient 37 year old came to the faculty of Dentistry: Complain of Chronic Halitosis, and his medical his story were finding normal beside gastritis and have the experience of influct gastric for a year, the main cause is:

Explanation

The correct answer is "A gas that comes from bacteria found on the surface of the tongue." This is because chronic halitosis, or bad breath, is often caused by the release of volatile sulfur compounds (VSCs) produced by bacteria on the tongue. These bacteria break down proteins and release VSCs, which have a foul odor. Gastritis and gastric reflux may contribute to bad breath, but they are not the main cause. Smoking and drinking alcohol can also contribute to bad breath, but they are not the main cause in this case.

Submit
99. A 4 year old, child presents with fever, gingival bleeding and extensive oral infection in his mouth. His medical history unremarkable. What would be you most likely diagnosis:

Explanation

Based on the symptoms described (fever, gingival bleeding, extensive oral infection), the most likely diagnosis is a Herpes Simplex infection. Herpes Simplex is a viral infection that commonly causes oral lesions and can lead to symptoms such as fever and gingival bleeding. The other options, such as Candidal infection, Herpes Zoster infection, Paramyxovirus infection, and Recurrent Aphthous Ulceration, do not fit the given symptoms as well as Herpes Simplex infection does.

Submit
100. A 60 Y old mal presents with unilateral facial paralysis, cutaneous lesions of external meatus hearing defects, and vertigo. What would be your most likely diagnosis:

Explanation

The symptoms described in the question, including unilateral facial paralysis, cutaneous lesions of the external meatus, hearing defects, and vertigo, are consistent with Ramsay Hunt syndrome. This syndrome is caused by the varicella-zoster virus, the same virus that causes chickenpox and shingles. It affects the facial nerve and can result in facial paralysis and other neurological symptoms. Bell's palsy is another condition that can cause facial paralysis, but it does not typically present with the additional symptoms described in the question. Melkerson Rosenthal syndrome is a rare disorder characterized by recurrent facial paralysis, facial swelling, and a fissured tongue. Trigeminal neuralgia is a condition characterized by severe facial pain, but it does not typically present with the other symptoms described in the question.

Submit
101. A 55 Y old edentulous female presents with an erythemathous patch on her palate. She wears complete dentures and has a history of Insulin Dependant Diabetes Mellitus. Youmark a diagnosis of denturs stomatitis. Which drug is most likely to be effective for her?:

Explanation

Miconazol is the most likely effective drug for the patient because denture stomatitis is a fungal infection commonly caused by Candida species. Miconazol is an antifungal medication that is effective against Candida, making it an appropriate treatment choice in this case. The other options, Augmentin, Hydrocortisone, Ibuprofen, and Phenytoin, are not indicated for the treatment of fungal infections.

Submit
102. A 35 Y old female presents with clicking in her right temporomandibular joint for the last six months. She does not have any pain or limitation of her mouth opening. Clinical examination does not reveal odontogenic cause. What does this condition most likely represent?:

Explanation

The patient's presentation of clicking in the temporomandibular joint without pain or limited mouth opening suggests internal derangement of the joint. Internal derangement refers to a disruption in the normal functioning of the joint, which can occur due to various factors such as disc displacement, joint hypermobility, or joint degeneration. In this case, the absence of pain or limitation in mouth opening rules out other conditions such as ankylosis, fracture, myofascial pain, or rheumatoid arthritis, making internal derangement the most likely explanation.

Submit
103. The characteristic appearance of the tongue in the Scarlet fever has been referred to as:

Explanation

The characteristic appearance of the tongue in Scarlet fever is referred to as "Strawberry tongue." This term is used to describe the swollen and red appearance of the tongue, which resembles a strawberry. It is a common symptom of Scarlet fever, caused by the streptococcus bacteria, and is often accompanied by other symptoms such as a red rash on the body.

Submit
104. Caused by an infection by normal commensal (endogenous) organisms of humans and does not require contact with an infected individual. the most common from of actinomycosis is :

Explanation

Actinomycosis is caused by normal commensal organisms that are normally present in the human body. It does not require contact with an infected individual to occur. Among the given options, cervicofacial actinomycosis is the most common form. This type of actinomycosis affects the head and neck region, specifically the jaw, mouth, and throat.

Submit
105. The etiology of the behcet’s syndrome is:

Explanation

Behcet's syndrome is a chronic autoimmune disorder characterized by inflammation of blood vessels throughout the body. The immune system mistakenly attacks healthy cells and tissues, leading to the symptoms associated with the syndrome. This is supported by the fact that individuals with Behcet's syndrome often have abnormal immune responses, such as increased levels of certain antibodies. Trauma, viral infection, and fungal infection are not known to be direct causes of Behcet's syndrome.

Submit
106. When can nicotine stomatitis regression be seen with cessation of irritation ?:

Explanation

Nicotine stomatitis is a condition characterized by inflammation and white spots on the roof of the mouth due to excessive tobacco use. When the irritation from tobacco use is stopped, the inflammation and white spots can regress. The correct answer of 14 days suggests that within two weeks of quitting tobacco use, the symptoms of nicotine stomatitis can start to improve.

Submit
107. A 58 Y old female presents with is the abnormal wearing away of tooth structure that is caused by a repetitive mechanical habit such as improper tooth brushing. What would be your most likely diagnosis?:

Explanation

Abrasion is the most likely diagnosis for this patient because it refers to the abnormal wearing away of tooth structure caused by repetitive mechanical habits such as improper tooth brushing. This aligns with the patient's presentation of tooth wear due to a mechanical habit. Attrition refers to the natural wearing down of tooth structure due to normal use, while bruxism refers to the grinding or clenching of teeth, usually during sleep. Bulimia, on the other hand, is an eating disorder that can cause tooth erosion due to frequent vomiting.

Submit
108. The patient 37 year old came to the dental clinic, he Complain of Chronic halitosis disease, and medical history were finding normal beside gastritis and have the experience of influct gastric for a year, Which the following the most likely to interview is a method of obtaining the history?

Explanation

The most likely method of obtaining the history would be to interview the patient. Since the patient is the one experiencing the symptoms and has the medical history, they would be the best source of information. The learner, practitioner, and instructor may also be involved in the process, but the patient would be the primary source of information in this case.

Submit
109. The most common viral pathogen cultured from oral infection in transplantation recipient is:

Explanation

The most common viral pathogen cultured from oral infection in transplantation recipients is the Herpes simplex virus. This virus is known to cause oral herpes, which presents as cold sores or fever blisters on or around the mouth. It is highly contagious and can be easily transmitted through direct contact with an infected individual. In transplantation recipients, who often have weakened immune systems, the risk of developing oral herpes is increased. Therefore, it is important to identify and treat this virus promptly to prevent complications.

Submit
110. The patient 23 year old came to the dental clinic, he Complain of color enamel his teeth. Which the following the most likely the mottled enamel is produced by:

Explanation

The most likely cause of the mottled enamel in the patient's teeth is fluorine. Fluorosis is a condition that occurs when there is excessive intake of fluoride during tooth development, leading to the formation of mottled enamel. This can happen due to drinking water with high fluoride content, excessive use of fluoride toothpaste, or exposure to other sources of fluoride. The other options, such as syphilis, febrile diseases, acids, and tuberculosis, are not typically associated with mottled enamel.

Submit
111. Dens in dents occurs most commonly in the:

Explanation

Dens in dente, also known as dens invaginatus, is a developmental anomaly where the enamel invaginates into the dental pulp. It most commonly occurs in the maxillary lateral incisors, although it can also be found in other teeth. This condition can lead to complications such as tooth sensitivity, increased risk of caries, and pulpitis. Therefore, the correct answer is Maxillary lateral incisors.

Submit
112. An abnormal disease in the flow of saliva is called:

Explanation

Xerostomia is the correct answer because it refers to an abnormal disease in the flow of saliva. It is characterized by dry mouth due to reduced or absent saliva production. Ptyalism, Trismus, Salivation, and Saladenitis are not correct because they do not specifically refer to an abnormal disease in the flow of saliva.

Submit
113. The most common malignancy found in the oral cavity is:

Explanation

Squamous cell carcinoma is the most common malignancy found in the oral cavity. This type of cancer arises from the squamous cells that line the oral cavity, including the lips, tongue, gums, and the lining of the cheeks. Risk factors for squamous cell carcinoma of the oral cavity include tobacco use, excessive alcohol consumption, poor oral hygiene, and human papillomavirus (HPV) infection. It typically presents as a non-healing ulcer or a red or white patch in the mouth. Early detection and treatment are crucial for improving the prognosis of patients with squamous cell carcinoma.

Submit
114. Head and neck examination to exam the:?

Explanation

The correct answer is "All answers." This means that during a head and neck examination, all aspects mentioned in the other answer choices should be examined, including hair, eyes, ears, nose, facial form, lips, lymph nodes, sinuses, oro-facial region, parotid glands, TMJ, and muscles of mastication.

Submit
115. Vesicles or Bullae of the mucosa membrane or skin are seen in all the following except:

Explanation

Vesicles or bullae are fluid-filled blisters that can occur on the mucosa membrane or skin. They are commonly seen in conditions such as herpes simplex, herpes zoster, pemphigus, and agranulocytosis. However, vesicles or bullae are not a characteristic feature of systemic lupus erythematosus (SLE). SLE primarily affects multiple organs and tissues, including the joints, skin, kidneys, and blood cells. While skin rashes are common in SLE, they typically present as a malar rash or discoid rash rather than vesicles or bullae.

Submit
116. The Peak incidence of gingivitis in children occurs at ages:

Explanation

The peak incidence of gingivitis in children occurs at ages 10-13 years. This is because during this period, children undergo hormonal changes which can lead to an increased susceptibility to gum inflammation. Additionally, at this age, children may also have poor oral hygiene habits and may not be effectively removing plaque from their teeth, further contributing to the development of gingivitis. Therefore, it is important to educate and encourage proper oral hygiene practices in children during this age range to prevent or manage gingivitis.

Submit
117. Intra oral examination, as a dentist, we should check on:?

Explanation

As a dentist, during an intra oral examination, it is important to check on all aspects of the oral cavity. This includes examining the oral mucosa and soft tissues for any abnormalities or signs of disease, assessing the bony structure for any issues such as bone loss or fractures, and examining the teeth and supporting structures for any signs of decay, gum disease, or other dental problems. By checking on all these aspects, a comprehensive assessment of the patient's oral health can be made, allowing for appropriate diagnosis and treatment planning.

Submit
118. Teeth and supporting structures except one:?

Explanation

The correct answer is Torus. A torus is a bony growth that can occur in the mouth, typically on the roof of the mouth or on the lower jaw. It is not directly related to teeth or their supporting structures such as the periodontium (which includes the gums, periodontal ligament, and alveolar bone) or the pulpe (the soft tissue inside the tooth). Therefore, the torus is the exception among the given options.

Submit
119. A patient present with a small, asymptomatic swelling in anterior palatal area with relation to maxillary central incisor, Radiography of the area show a pear shaped radiolucency between the root of canal incisor. Both the central incisor are non-carious, vital and there is no history of trauma to them. The most likely diagnosis is:

Explanation

The patient's presentation of a small, asymptomatic swelling in the anterior palatal area with a pear-shaped radiolucency between the root of the maxillary central incisors suggests the most likely diagnosis of a Nasopalatine cyst. This type of cyst typically occurs in the midline of the palate and is often discovered incidentally on radiographs. It is usually asymptomatic and can present as a small swelling. The fact that both central incisors are non-carious, vital, and without a history of trauma further supports this diagnosis. Radicular cysts, globulomaxillary cysts, and nasoalveolar cysts are less likely based on the given information.

Submit
120. Prehypertension artery, the systolic and diastolic from:?

Explanation

This answer indicates that prehypertension is defined by a systolic blood pressure reading between 120-139 mmHg and a diastolic blood pressure reading between 80-89 mmHg.

Submit
121. This test measures all of the cholesterol in all the lipoprotein particles, is the:?

Explanation

This test measures all of the cholesterol in all the lipoprotein particles, including high-density lipoprotein (HDL), low-density lipoprotein (LDL), very low-density lipoprotein (VLDL), and triglyceride.

Submit
122. A patient present with hairy tongue, there is most likely hypertrophy of:

Explanation

Hairy tongue is a condition characterized by an overgrowth or elongation of the filiform papillae on the surface of the tongue. These papillae are responsible for the rough texture of the tongue and can become stained or discolored, giving the appearance of "hair". The other types of papillae mentioned (fungiform, foliate, and circumvallate) do not typically contribute to the development of hairy tongue. Therefore, the most likely cause of hairy tongue in this patient is hypertrophy of the filiform papillae.

Submit
123. The hemostasis: platelet plug and Seconded hemostasis : the coagulation cascade, Vasculature maintenance, Reservoir for soluble factor, is the function of the:?

Explanation

Platelets play a crucial role in hemostasis, which is the process of stopping bleeding. They form a platelet plug at the site of injury to prevent further blood loss. Additionally, platelets are involved in the coagulation cascade, which leads to the formation of a fibrin clot to seal the injured blood vessel. Platelets also help in maintaining the integrity of blood vessels, act as a reservoir for soluble factors involved in clotting, and contribute to the overall function of hemostasis. Therefore, the correct answer is Platelets.

Submit
124. Site most commonly affected by basal cell carcinoma is:

Explanation

Basal cell carcinoma is a type of skin cancer that commonly affects the skin of the middle third of the face. This area includes the nose, cheeks, and forehead. Basal cell carcinoma is often caused by long-term sun exposure, and these areas of the face are more prone to sun damage. Therefore, it is important to protect the skin in this area from the harmful effects of the sun to reduce the risk of developing basal cell carcinoma.

Submit
125. The chef complain should be recorded in the:?

Explanation

The correct answer is "All answers" because when recording a chef's complaint, it is important to capture it in the patient's own words, mark it in quotation marks to indicate it is a direct quote, and include it in the master problem list for comprehensive documentation. By choosing "All answers," the answer encompasses all the appropriate methods for recording the chef's complaint accurately and thoroughly.

Submit
126. The clinical presentation of the symptoms of tonsillitis by Virus, The most likely present is:

Explanation

The correct answer is "Red swollen tonsils, Throat redness". This is the most likely presentation of tonsillitis caused by a virus. Tonsillitis is an inflammation of the tonsils, which are located at the back of the throat. Red, swollen tonsils and throat redness are common symptoms of tonsillitis. The other options, such as swollen uvula, whitish spots, and furry tongue, may be present in other conditions but are not typically associated with tonsillitis caused by a virus.

Submit
127. The reasons of tooth ache or Pain after dental filling is the:?

Explanation

The correct answer is "Chief complain". This is because the chief complaint refers to the main reason why the patient sought dental treatment in the first place. In the context of toothache or pain after a dental filling, the chief complaint would provide valuable information about the specific symptoms experienced by the patient, allowing the dentist to diagnose and address the issue effectively.

Submit
128. The final diagnosis for the dentist decision depend on the ?

Explanation

The final diagnosis for the dentist's decision depends on the blood test and biopsy. These two procedures are crucial in determining the presence of any abnormalities or diseases in the patient's oral health. Blood tests can provide valuable information about the patient's overall health and identify any underlying conditions that may affect their oral health. Biopsy, on the other hand, involves taking a sample of tissue from the affected area for further examination under a microscope, which can help in diagnosing oral diseases such as oral cancer. Therefore, both blood tests and biopsies are essential in making an accurate diagnosis and determining the appropriate course of treatment.

Submit
129. Way to success diagnosis depend on clear the:?

Explanation

The way to success in diagnosis depends on both history taking and examination. History taking involves gathering information about the patient's symptoms, medical history, and any relevant factors that may contribute to their condition. This helps the healthcare professional to understand the context and potential causes of the patient's symptoms. Examination involves physically assessing the patient, which may include a physical examination, laboratory tests, or imaging studies. By combining the information obtained from history taking and examination, healthcare professionals can make an accurate diagnosis and develop an appropriate treatment plan.

Submit
130. The method that can completely cured from illness for getting healthy, it is the:?

Explanation

The correct answer is Treatment. Treatment refers to the methods or procedures used to cure or alleviate an illness or health condition. It involves the administration of medications, therapies, surgeries, or other interventions that are specifically designed to address the underlying cause of the illness and restore the individual's health. Treatment aims to eliminate or reduce symptoms, prevent complications, and ultimately achieve a state of complete recovery or improvement in the person's overall well-being.

Submit
131. Types of treatment:?

Explanation

The question is asking about the types of treatment, and the correct answer is "All answers." This means that all of the options listed (Physiology, Psychology, Diet, Drug, surgery, and Radiotherapy) are types of treatment.

Submit
132. A medical care that involves the use of medications, either alone or in combination with other types of therapy is the:?

Explanation

Pharmacological treatment refers to the medical care that involves the use of medications, either alone or in combination with other types of therapy. This type of treatment focuses on using drugs to manage and alleviate symptoms, treat diseases, or prevent the progression of a condition. It is a common approach in healthcare and can be effective in addressing various medical issues. Other options such as drug and surgery treatment, prevention treatment, and dietary treatment may also be used in certain cases, but pharmacological treatment specifically emphasizes the use of medications.

Submit
133. The supportive and motivate of patients more effective than using medication is the:

Explanation

The given question states that the supportive and motivational approach towards patients is more effective than using medication. This implies that psychological factors play a significant role in the well-being and recovery of patients. Therefore, the answer "Psychology" is the most appropriate choice as it aligns with the importance of mental and emotional support in patient care.

Submit
134. A medical care that involves the use of medications, either alone or in combination with other types of therapy is ?:

Explanation

Pharmacology is the study of drugs and their effects on the body. It involves the use of medications to treat diseases and conditions. In medical care, pharmacology plays a crucial role as it helps in understanding how drugs interact with the body and how they can be used to alleviate symptoms or cure illnesses. It can be used alone or in combination with other types of therapy to provide effective treatment.

Submit
135. Which one of the following is the most common site of occurrence of Oral Squamous Cell Carcinoma:

Explanation

Oral Squamous Cell Carcinoma is most commonly found on the lateral border of the tongue. This is because the tongue is a common site for the development of oral cancers, and the lateral border is particularly susceptible due to its exposure to various irritants such as tobacco and alcohol. Additionally, the lateral border of the tongue has a higher number of blood vessels and lymph nodes, which can contribute to the spread of cancer cells. Therefore, it is crucial to regularly examine this area for any signs of abnormality or cancerous growth.

Submit
136. The cause of oral lichen planus is ?:

Explanation

The cause of oral lichen planus is unknown. This means that the exact reason why someone develops this condition is not yet understood. It is believed to be an autoimmune disorder, where the immune system mistakenly attacks the cells of the oral mucosa. However, the specific triggers or factors that lead to this immune response are still unclear. Researchers continue to study this condition in order to determine its cause and develop effective treatments.

Submit
137. Denture sore mouth is:

Explanation

Denture sore mouth refers to a condition where the mouth becomes sore and uncomfortable due to wearing dentures. It can be caused by a candidial infection, which is a type of fungal infection. It is more commonly seen in individuals wearing upper dentures. One of the symptoms experienced by individuals with denture sore mouth is a burning sensation in the mouth. Therefore, all the given options are true statements about denture sore mouth.

Submit
138. Aphthous stomatitis are ulcers in the mouth that usually occurs on 3 places, except:

Explanation

Aphthous stomatitis refers to ulcers in the mouth that commonly appear on the cheeks, tongue, and inside the lips. The hard palate, which is the roof of the mouth, is not typically affected by these ulcers.

Submit
139. Which of the following statement is wrong in relation to denture stomatitis:

Explanation

Denture stomatitis is a condition characterized by inflammation of the oral mucosa, often caused by wearing dentures. The given statement that is wrong in relation to denture stomatitis is "More common in man." Denture stomatitis is actually more common in women than in men. This condition can occur in both genders, but studies have shown a higher prevalence in women.

Submit
140. The alveolar osteitis is a complication of having tooth extracted. Occurs in about 0.5-5% of routine extraction and 25-30% of extraction of impacted mandibular third molar. Pain is often resistant to common analgesic, delayed healing time. Which one the following meaning of the:

Explanation

Dry socket is the correct answer. Alveolar osteitis, commonly known as dry socket, is a painful condition that occurs after a tooth extraction. It is characterized by delayed healing, severe pain, and exposed bone in the socket where the tooth was removed. The pain associated with dry socket is often resistant to common painkillers. This condition is more common in impacted mandibular third molar extractions and can occur in about 0.5-5% of routine extractions. Osteomyelitis refers to an infection of the bone, osteitis refers to inflammation of the bone, and apthous ulcer is a type of mouth ulcer, none of which accurately describe the given scenario.

Submit
141. 48 year old male reported with the chief complaint of ulcer on the gums and on general examination, headache, fever and malaise was observed. On oral examination, sharply punched out crater-like erosions of the interdental papillae of sudden onset were noted. The patient also complained of bad breath. What do you think is the most likely diagnosis?:

Explanation

The patient's symptoms of ulceration on the gums, headache, fever, malaise, and bad breath are consistent with Acute Necrotizing Ulcerative Gingivitis (ANUG). ANUG is a severe form of gingivitis characterized by painful, punched-out ulcerations of the interdental papillae. It is often associated with poor oral hygiene, stress, smoking, and immunosuppression. Chediak Higashi disease is a rare genetic disorder that affects the immune system and causes recurrent infections, but it does not present with the specific oral symptoms described. Desquamative gingivitis is a clinical sign seen in various mucocutaneous diseases, but it does not typically present with punched-out ulcerations. Pericoronitis is inflammation of the soft tissues around a partially erupted tooth, which does not explain the patient's symptoms.

Submit
142. Common variation of a normal tongue the:

Explanation

The correct answer is "All answers" because all three options (Scalloped tongue, Hairy tongue, Fissured tongue) are common variations of a normal tongue. Scalloped tongue refers to a condition where the edges of the tongue appear indented or scalloped, usually due to pressure from the teeth. Hairy tongue is a condition where the papillae on the surface of the tongue become elongated, giving it a hairy appearance. Fissured tongue is characterized by deep grooves or cracks on the surface of the tongue.

Submit
143. The growth of odontogenic keratocyst mainly takes place in the…..direction:

Explanation

The growth of odontogenic keratocyst mainly takes place in the antero-posterior direction. This means that the cyst grows from the front to the back of the mouth.

Submit
144. Classification of the secondary oral thrush (group II) like:

Explanation

The correct answer is Chronic multifocal oral candidiasis. This is because the question is asking for the classification of secondary oral thrush in group II, and Chronic multifocal oral candidiasis is one of the classifications mentioned in the given options.

Submit
145. A 55 Y old female presents with a 6 mouth of history recurrence mealtime swelling and pain in right submandibular region. Clinical examination does not reveal any dental abnormal. What is the most likely cause of her symptoms?:

Explanation

Sialolithiasis is the most likely cause of the patient's symptoms. Sialolithiasis refers to the formation of stones (calculi) in the salivary glands or ducts, obstructing the flow of saliva. The patient's history of recurrent swelling and pain in the right submandibular region, along with the absence of dental abnormalities, is consistent with the diagnosis of sialolithiasis. Lymphadenitis refers to inflammation of the lymph nodes and is not the most likely cause in this case. Sialometaplasia is a benign condition characterized by the transformation of salivary gland tissue, but it does not typically present with recurrent swelling and pain. Sialorrhea refers to excessive saliva production, and xerostomia refers to dry mouth, neither of which are consistent with the patient's symptoms.

Submit
146. Dry socket is a form of :

Explanation

Dry socket is a condition that occurs after a tooth extraction when the blood clot in the socket dissolves or dislodges, leaving the bone and nerves exposed. This leads to severe pain and delayed healing. Periostitis is the inflammation of the periosteum, which is the outer layer of the bone. In the case of dry socket, the inflammation affects the periosteum around the socket, causing periostitis. Therefore, the correct answer is periostitis.

Submit
147. Inflammation of the lips is referred to as:

Explanation

Cheilitis refers to inflammation of the lips. This condition can be caused by various factors such as dryness, allergies, infections, or exposure to irritants. It is characterized by redness, swelling, cracking, and sometimes pain or discomfort in the lips. Cheilitis can be acute or chronic and may require treatment depending on the underlying cause. Stomatitis, glossitis migrans, Vincent Angina, and sialadenitis are all different conditions that involve inflammation, but they specifically refer to inflammation of the mouth, tongue, throat, or salivary glands, respectively.

Submit
148. A 70 year old gardener presents with crusting of lower lip for the last 4-5 years which gets worse in the cold weather. It is diagnosed as actinic cheilitis which shows significant dysplasia on histopathology. Its management is:

Explanation

The correct management for actinic cheilitis with significant dysplasia on histopathology is vermilionectomy or laser ablation, sun avoidance, and the use of sunscreens. Actinic cheilitis is a precancerous condition caused by long-term sun exposure, and significant dysplasia indicates a higher risk of progression to squamous cell carcinoma. Vermilionectomy or laser ablation can remove the affected tissue and reduce the risk of malignant transformation. Sun avoidance and the use of sunscreens with high sun protection factor (SPF) are important to prevent further damage from sun exposure. Advising the patient to apply Vaseline or stating that no management is required would not address the underlying dysplasia or reduce the risk of progression to cancer.

Submit
149. Infectious in oral diseases is a :

Explanation

Infectious in oral diseases can be caused by bacteria, fungi, and viruses. Bacterial infections such as dental caries and periodontal diseases are common in the oral cavity. Fungal infections like oral thrush can also occur, especially in individuals with weakened immune systems. Viral infections such as herpes simplex virus can cause oral ulcers and cold sores. Therefore, all three options (bacterial, fungal, and viral) are correct as they can all contribute to infectious oral diseases.

Submit
150. A patient presents with soreness and bleeding of gingivae, necrosis of the gingival papillae and marked halitosis. T]\]he drug of choice in this case would be:

Explanation

The patient is presenting with symptoms of severe gum disease, including soreness, bleeding, necrosis, and halitosis. Metronidazole is an antibiotic that is effective against the bacteria that cause gum infections. The use of chlorhexidine mouthwash can further help in reducing the bacterial load in the mouth. Therefore, the combination of Metronidazole200mg T.D.S with chlorhexidine mouthwash is the most appropriate treatment option for this patient.

Submit
151. Acute pseudomembranous candidiasis on denture-bearing mucosa of the hard palate:

Explanation

The correct answer is "The white plaque rub off." This suggests that the white plaque can be easily removed from the denture-bearing mucosa of the hard palate. This characteristic is indicative of acute pseudomembranous candidiasis, a fungal infection that can occur in individuals wearing dentures. The ability to rub off the plaque distinguishes it from other types of lesions that would not be easily removable.

Submit
152. A 51 year old Jewish man presents with a 9 months history of mouth ulceration causing discomfort in eating. He reports of large blisters inside his lips that rapidly burst. He has also developed skin lesions on his back & eroded skin can be seen clinically. There are no ocular or genital symptoms. Nikolsky sign is positive. The investigation to make definitive diagnosis include:

Explanation

The correct answer is Direct immunofluorescence (IMF) of peri-lesional tissue and indirect immunofluorescence (IMF) of blood serum. This is because the patient's presentation with mouth ulceration, skin lesions, and positive Nikolsky sign is suggestive of a condition called pemphigus vulgaris, an autoimmune blistering disorder. Direct IMF of peri-lesional tissue can help confirm the presence of IgG antibodies and complement deposition at the site of blister formation, while indirect IMF of blood serum can detect circulating autoantibodies against desmoglein 1 and 3, which are specific to pemphigus vulgaris. These investigations are necessary to make a definitive diagnosis.

Submit
153. Desquamative gingivitis cause by:

Explanation

Desquamative gingivitis refers to a condition characterized by inflammation and peeling of the gums. Erythema multiforme is a type of hypersensitivity reaction that can cause desquamative gingivitis. It is characterized by the presence of target-like skin lesions and can affect the oral mucosa, including the gums. Pemphigus vulgaris and pemphigoid are autoimmune blistering disorders that can also cause desquamative gingivitis, but they are not the correct answer in this case. Mucosa burns are not a known cause of desquamative gingivitis.

Submit
154. The term of tumor which is a benign composed of adipose tissue. It’s the most common benign form of soft tissue tumor, which one?:

Explanation

A lipoma is a benign tumor composed of adipose tissue. It is the most common benign form of soft tissue tumor. Therefore, the correct answer is Lipoma.

Submit
155. When the body tissues undergo a disease process, certain types of condition develop. Which term correctly describes that which appears as an abnormal fluid-filled sac within the tissues?:

Explanation

A cyst is a term used to describe an abnormal fluid-filled sac that develops within the body tissues during a disease process. This can occur in various parts of the body and can be caused by different factors such as infections, blockages, or genetic conditions. Cysts can vary in size and can sometimes cause discomfort or other symptoms depending on their location and size.

Submit
156. In the first months of life, babies often develop resistance to specific pathogenic micro-organisms by inheriting it from their mother. Which of the following terms is used to describe this condition?:

Explanation

Passive immunity is the correct term to describe the condition where babies develop resistance to specific pathogenic micro-organisms by inheriting it from their mother. This is because the mother transfers antibodies to the baby through the placenta or breast milk, providing temporary protection against certain infections. Acquired immunity refers to the immune response developed by an individual after exposure to a pathogen or through vaccination. Dead vaccine and live vaccine refer to different types of vaccines, while natural immunity refers to the immune response developed by an individual without any external intervention.

Submit
157. Treatment management of the Hand food mouth disease is :

Explanation

Hand, foot, and mouth disease is a viral infection that primarily affects children. It is usually a self-limiting illness, meaning it resolves on its own without specific medical treatment. The symptoms, such as fever, sore throat, and blisters on the hands, feet, and mouth, can be managed with over-the-counter pain relievers and fever reducers. It is important to maintain good hygiene practices, such as frequent handwashing, to prevent the spread of the virus. However, there is no specific antiviral or antibiotic treatment for hand, foot, and mouth disease.

Submit
158. In military tuberculosis, dissemination of microorganism occur via:

Explanation

In military tuberculosis, the dissemination of the microorganism occurs through the bloodstream. This means that the bacteria responsible for tuberculosis can enter the bloodstream and spread to different parts of the body, causing infection in various organs and tissues. The bacteria can be transported to distant sites through the blood, leading to the development of extra-pulmonary tuberculosis. The lymphatics, saliva, and pulmonary fluid may play a role in the transmission of tuberculosis in certain cases, but the primary mode of dissemination in military tuberculosis is through the bloodstream.

Submit
159. What histological form is most common to be found in ameloblastoma ?:

Explanation

The most common histological form found in ameloblastoma is the follicular variant. This variant is characterized by the presence of well-defined islands or nests of odontogenic epithelium that resemble the early stages of tooth development. These islands are surrounded by a fibrous connective tissue stroma. The follicular variant is the most frequently encountered subtype of ameloblastoma and is associated with a relatively better prognosis compared to other variants.

Submit
160. Mary 36 year old came to the dental clinic, he Complain of the Pericoronitit of the molar 38 and the dental hisstory were finding normal and have the experience of bleeding gum and painfully for two week, Which the following the most likely to interview is the chief complaint/history taking focus on the Pain except is the:?

Explanation

The correct answer is "Palpation and precaution during visit." This answer is the most likely focus of the chief complaint/history taking because it addresses the immediate concerns of the patient during their visit to the dental clinic. Palpation refers to the examination of the affected area through touch, which can help determine the extent of the problem and any potential complications. Precaution during the visit is also important to ensure the patient's safety and to avoid aggravating their condition. Therefore, this option is the most relevant in addressing the patient's pain and discomfort during the visit.

Submit
161. 25 year old of the women to come the hospital every year for the medical and dental check up, the list of the Medical history is the information about certain medical conditions for the dental management, which one most likely in the condition my affective during to the ?

Explanation

The correct answer is "Dental practices." Based on the information provided, the question is asking which condition is most likely to affect the dental management of the patient. The term "Dental practices" refers to the procedures and protocols followed by dentists in providing dental care. Therefore, it is logical to assume that the condition affecting dental management would be related to the practices followed by dentists.

Submit
162. The cyst occurring under the tongue, cause by obstruction of a salivary gland duct, is called a:

Explanation

A cyst occurring under the tongue, caused by the obstruction of a salivary gland duct, is called a ranula. A ranula is a type of mucocele that forms when the duct of the sublingual or submandibular salivary gland becomes blocked, leading to the accumulation of saliva. This causes a cystic swelling or bump under the tongue.

Submit
163. The term of the oral examination is the exposed?

Explanation

The correct answer is a combination of two options: "Surface structure of the head neck and face" and "Detailed examination of the oral cavity, dentition, oropharynx." This suggests that the term of the oral examination involves both assessing the external features of the head, neck, and face, as well as conducting a thorough examination of the oral cavity, teeth, and oropharynx. The option "Detailed examination of lingual for the mastication and specking" is not included in the correct answer, indicating that it is not relevant to the term of the oral examination.

Submit
164. The clearance of Examination of Dentistry to : ?

Explanation

The correct answer is "All answers." This means that all of the options provided are correct explanations for the clearance of the Examination of Dentistry. The clearance is necessary to elicit other conditions, have a specific diagnosis, and assist the dentist in making an appropriate and sufficient treatment plan.

Submit
165. There are three basic rules for every clinical examination except one:?

Explanation

The correct answer is "General before simple." This means that in a clinical examination, the general aspects of a patient's condition should be assessed before focusing on specific or simple aspects. This rule ensures that a comprehensive evaluation is conducted and potential underlying issues are not overlooked.

Submit
166. Oral cytological smear are of no value in the diagnosis of:

Explanation

Oral cytological smears are a valuable tool in diagnosing various conditions, including oral cancer, primary intraoral herpes simplex, recurrent intraoral herpes simplex, and herpes zoster. However, lipoma is a benign tumor composed of fat cells, and it can be diagnosed through clinical examination and imaging techniques rather than cytological smears. Therefore, oral cytological smears are of no value in the diagnosis of lipoma.

Submit
167. A patient present with a loss of tooth structure due to mechanical action of a foreign element, such as a hard bristle toothbrush or a lip piercing. This can occur anywhere that the foreign object rubs on the teeth. The most likely a tooth of the diagnosis is:

Explanation

Abrasion is the most likely diagnosis in this case because it refers to the loss of tooth structure due to mechanical action of a foreign element. In this scenario, the foreign element could be a hard bristle toothbrush or a lip piercing, both of which can cause friction and wear away the tooth structure. This can occur anywhere the foreign object rubs on the teeth, leading to abrasion. Attrition refers to the natural wearing down of tooth structure from normal use, erosion refers to the loss of tooth structure due to chemical processes, and abfraction refers to the loss of tooth structure at the gum line due to excessive forces.

Submit
168. A patient presented with asymptomatic, smooth, circumscribed red area in midline anterior to circumvallate papillae on the dorsum of tongue with microscopic evidence of epithelial hyperplasia. The most probable diagnosis of the condition is:

Explanation

The patient's presentation of a smooth, circumscribed red area in the midline anterior to the circumvallate papillae on the dorsum of the tongue, along with microscopic evidence of epithelial hyperplasia, is consistent with the diagnosis of median rhomboid glossitis. This condition is characterized by a flat or slightly elevated red patch in the midline of the tongue, which is caused by chronic inflammation and fungal infection. Geographic tongue, hairy tongue, and lingual thyroid do not match the patient's clinical findings and microscopic evidence.

Submit
169. A patient present with an asymptomatic soft, fluctuant swelling of the angle of mandible anterior to sternocleido-mastoid muscle which he stated to be present since his childhood days. Regional lymph nodes are nonpalpable with normal radiographic pictures of the area and normal blood and urine examination. Aspiration of swelling shows yellow-brown fluid. Most probable diagnosis in the condition is:

Explanation

The most probable diagnosis in this case is a branchial cleft cyst. This is supported by the patient's history of the swelling being present since childhood, the location of the swelling at the angle of the mandible anterior to the sternocleidomastoid muscle, and the characteristics of the fluid obtained from aspiration. Additionally, the absence of palpable lymph nodes and normal radiographic pictures, blood, and urine examination further support the diagnosis of a branchial cleft cyst.

Submit
170. The Teeth and supporting structures and Bony structure is intra oral examination except one:?

Explanation

The question is asking which option is not included in an intraoral examination of the teeth and supporting structures. Intraoral examinations involve examining the oral cavity and its structures, such as the teeth, gums, and bones. Blood tests and biopsies, however, are not part of an intraoral examination as they involve analyzing blood samples and taking tissue samples for further analysis. Therefore, the correct answer is "Blood test, Biopsy."

Submit
171. A 25-year old patient with an infection and swelling, which are oval shaped masses of lymph gland tissue located on both sides of the back of the throat, this is inflammation, It’s usually caused by viral infection or, less commonly, a bacterial. The most likely diagnosis is:

Explanation

The patient's symptoms of infection and swelling, along with the description of oval-shaped masses of lymph gland tissue located on both sides of the back of the throat, are consistent with tonsillitis. Tonsillitis is an inflammation of the tonsils, which are lymph nodes located at the back of the throat. It is commonly caused by viral infections, although bacterial infections can also be a cause. Therefore, the most likely diagnosis for this patient is tonsillitis.

Submit
172. The time of onset, Mode of onset, the site, changes in intensity/quality, severity of complaint is the:?

Explanation

The correct answer is "History of the chief complaint." This is because the question is asking about the various factors related to the chief complaint, such as the time and mode of onset, the site, changes in intensity/quality, and severity of the complaint. These factors are important in understanding the patient's condition and providing appropriate treatment.

Submit
173. The diagnosis is identify the nature and cause of a disease or injury through the?

Explanation

The correct answer is "Examination and review of laboratory data and Biopsy." This is because the diagnosis of a disease or injury involves conducting a thorough examination of the patient, which includes reviewing laboratory data such as blood tests, imaging tests, and other diagnostic tests. Additionally, a biopsy may be necessary to obtain a sample of tissue for further analysis and confirmation of the diagnosis.

Submit
174. A 28-year old patient with present the conditions that cause red, swollen patches in the Corners of you mouth, where the lips meet and make an angle. The most likely diagnosis is:

Explanation

The patient's symptoms of red, swollen patches in the corners of the mouth where the lips meet and make an angle are consistent with Angular Cheilitis. This condition is characterized by inflammation and cracking at the corners of the mouth, often caused by factors such as moisture, yeast or bacterial infections, or nutritional deficiencies. Stomatitis allergic, herpes simplex infection, and traumatic ulcer are less likely diagnoses based on the given symptoms.

Submit
175. The etiology of verucca vulgaris is:

Explanation

Verucca vulgaris is caused by a viral infection. It is a common skin condition caused by the human papillomavirus (HPV). The virus enters the skin through small cuts or abrasions and causes the formation of warts, which are characterized by their rough texture and cauliflower-like appearance. The viral infection can spread through direct contact with an infected person or by touching surfaces that have come into contact with the virus. Treatment options for verucca vulgaris include topical medications, cryotherapy, and surgical removal.

Submit
176. Refers to check-ups (examinations and cleanings) and minor restorative treatment. Moreover, to intercept major complication?:

Explanation

The correct answer is "Prevention" because the given statement is referring to check-ups, cleanings, and minor restorative treatment, which are all part of preventive dental care. The statement also mentions intercepting major complications, which further supports the idea of prevention.

Submit
177. Leukoplakia is whitish path or plaque which is:

Explanation

Leukoplakia is a whitish path or plaque that may occur due to smoking tobacco. This condition is not always reversible and is not caused by an infection or a congenital disease. Smoking tobacco is a common risk factor for developing leukoplakia, as the chemicals in tobacco can irritate the lining of the mouth and lead to the formation of white patches. Therefore, the correct answer is "May due to smoking tobacco".

Submit
178. The main cause of the Halitosis is?:

Explanation

Tooth decay and gum disease are the main causes of Halitosis. These oral health issues can lead to the buildup of bacteria in the mouth, which produces unpleasant odors. When teeth are not properly cleaned and maintained, plaque and tartar can form, causing tooth decay and gum disease. This can result in bad breath. Regular brushing, flossing, and dental check-ups can help prevent these conditions and reduce the risk of Halitosis.

Submit
179. A 65 year old lady with ill fitting dentures and a very poor oral hygiene under treatment for Addison’s disease has generalized inflamed mucosa with some white patches. The blood tests are normal. It is suggestive of:

Explanation

The presence of ill-fitting dentures and poor oral hygiene in a 65-year-old lady, along with generalized inflamed mucosa and white patches, suggests candidiasis. Candidiasis is a fungal infection caused by Candida species, commonly seen in individuals with compromised immune systems or those wearing dentures. The normal blood tests indicate that the infection is localized to the oral cavity rather than systemic. Lichen planus, leukoplakia, and oral mucosa fibrosis may also present with similar symptoms, but the given clinical history points more towards candidiasis.

Submit
180. Xerostomia may occur due to any of the following except:

Explanation

This question is asking for the cause of xerostomia, or dry mouth, and states that all of the given options may cause it except for "None of answers." This means that none of the options listed are a cause of xerostomia. Therefore, the correct answer is "None of answers."

Submit
181. Target lesion are seen in:

Explanation

Target lesions are a characteristic feature of erythema multiforme. These lesions appear as concentric rings with a dusky center and a pale outer ring. They are typically seen on the extremities, particularly the hands and feet. Erythema multiforme is a hypersensitivity reaction that can be triggered by infections, medications, or other factors. Pemphigoid and lichen planus do not typically present with target lesions, making erythema multiforme the correct answer.

Submit
182. Leukoplakia very often precedes the development of a:

Explanation

Leukoplakia is a condition characterized by white patches or plaques on the mucous membranes, commonly found in the mouth. These patches are often precancerous and can progress to the development of a malignant tumor, such as oral cancer. Therefore, the correct answer is a malignant tumor.

Submit
183. A boy shows pearly white thickening of the buccal mucosa and has been present for some years. His younger brother also has the same condition. The condition is most likely to be:

Explanation

White sponge nevus is a rare genetic disorder characterized by thickening of the buccal mucosa, giving it a pearly white appearance. It is usually present from birth or develops in early childhood. The fact that both the boy and his younger brother have the same condition suggests a genetic inheritance pattern. Lichen planus, leukoplakia, and submucous fibrosis are other conditions that can cause changes in the buccal mucosa, but they do not typically present with the same pearly white thickening seen in white sponge nevus.

Submit
184. Size of the minor ulcers is:

Explanation

The correct answer is "Less than 10mm and heal within 1-2 weeks." This means that the size of the minor ulcers is smaller than 10mm and they typically heal within a period of 1-2 weeks.

Submit
185. Below are the causes of Aphthous stomatitis, except:

Explanation

The given answer is "Smoking." Aphthous stomatitis, also known as canker sores, is commonly caused by factors such as stress and lack of sleep, poor nutrition, and trauma to the mouth from self-inflicted bites. However, smoking is not typically considered a direct cause of aphthous stomatitis.

Submit
186. Garre’s osteomyelitis is a type of chronic osteomyelitis where there is:

Explanation

Garre's osteomyelitis is characterized by periosteal bone formation due to mild infection or irritation. This means that in response to the infection or irritation, the periosteum (the outer layer of the bone) starts forming new bone. This can be seen on imaging studies as thickening or elevation of the periosteum. This finding helps differentiate Garre's osteomyelitis from other types of chronic osteomyelitis. The other options mentioned in the question, such as diffuse cotton wool sclerosis and necrosis of bone with suppuration, are not specific to Garre's osteomyelitis and can be seen in other types of osteomyelitis as well.

Submit
187. A fungal infection of the mouth. It’s not contagious and usually successfully treated with antifungal medication, caused by group of yeasts called candida:

Explanation

Oral thrush is a fungal infection of the mouth caused by a group of yeasts called candida. It is not contagious and can be successfully treated with antifungal medication. The other options, such as oral leukoplakia, oral lichen planus, and oral ulcerative condition, do not specifically refer to a fungal infection and are not caused by candida. Therefore, the correct answer is oral thrush.

Submit
188. Which odontogenic cyst manifests characteristically as a pericoronal radiolucency of impacted molars? :

Explanation

The dentigerous cyst is an odontogenic cyst that is commonly associated with impacted molars. It is characterized by a pericoronal radiolucency, which means that it appears as a radiolucent area around the crown of an impacted tooth. This cyst forms when fluid accumulates between the crown of an unerupted tooth and the surrounding dental follicle. It is important to diagnose and treat dentigerous cysts to prevent complications such as infection or damage to adjacent structures.

Submit
189. Diseases and conditions make more susceptible to oral thrush like:

Explanation

The correct answer is "All answers". This means that all of the listed conditions and factors can make a person more susceptible to oral thrush. Taking inhaled corticosteroid medication for asthma, taking antibiotics for a long period or in high doses, having a vaginal yeast infection, having diabetes, being HIV/AIDS positive, and having cancer can all weaken the immune system and create an environment in the mouth that is favorable for the growth of the Candida fungus, which causes oral thrush.

Submit
190. Which of the following type of leukoplakia is more prone to become squamous cell carcinoma?:

Explanation

Proliferative verrucous leukoplakia is more prone to become squamous cell carcinoma because it is a progressive form of leukoplakia that has a higher risk of malignant transformation compared to other types. This type of leukoplakia is characterized by multiple lesions that grow and spread over time, making it more likely to develop into squamous cell carcinoma.

Submit
191. Classification of the primary oral thrush (group I) like:

Explanation

The correct answer includes the types of primary oral thrush, which are pseudo-membranous, erythema, and hyperplastic. It does not include the other two options, chronic multifocal oral candidiasis and chronic mucocutaneous candidiasis.

Submit
192. The name geographic tongue:

Explanation

The term "geographic tongue" is used to describe a condition where the tongue develops multiple areas of desquamation (loss) of the filiform papillae. These areas are irregularly shaped but well-demarcated and resemble a map. Over time, the smooth areas and the whitish margins of the tongue seem to migrate across its surface by healing on one border and extending on another. This description applies to all the given answers.

Submit
193. The usual oral mucosal reactions to the presence of a sensitizing the allergic is:

Explanation

The correct answer is "Erythema and Edema." Erythema refers to redness of the oral mucosa, which is a common reaction to allergens. Edema, on the other hand, refers to swelling of the oral mucosa, which can also occur as a result of an allergic reaction. Therefore, the usual oral mucosal reactions to the presence of a sensitizing allergen are erythema and edema.

Submit
194. A 30 year old male presents with a painful ulceration on the gingiva adjacent to carious lower right 5 for the last one week. He has been placing aspirin as a remedy for toothache in this carious tooth. The management of this patient includes:

Explanation

The management of this patient includes cessation of aspirin placement and application of topical benzydamine and chlorhexidine preparations. This is because the patient has developed a painful ulceration on the gingiva due to placing aspirin as a remedy for toothache in a carious tooth. Aspirin, when placed directly on the gingiva, can cause chemical burns and ulcerations. Therefore, the first step in managing this patient is to stop placing aspirin on the affected area. Topical benzydamine and chlorhexidine preparations can be applied to help alleviate pain and promote healing.

Submit
195. Treatment of the halitosis except one:

Explanation

The correct answer is "Provide antibiotic antiseptic". This is because halitosis, or bad breath, is commonly caused by poor oral hygiene, dental problems, or tongue coatings. Antibiotic antiseptics are not typically used as a primary treatment for halitosis. Instead, the other options listed, such as restoration of fillings, examination and cleaning of dental prostheses, daily oral hygiene measures, and tongue cleaning, are more effective in addressing the underlying causes of halitosis.

Submit
196. Site affected of the Burning Mouth Syndrome :

Explanation

The correct answer is "All answers." This means that all mucosa in the mouth, including the tongue, lips, and roof of the mouth, can be affected by Burning Mouth Syndrome. This condition causes a burning sensation in the mouth, which can be quite uncomfortable. It is important to seek medical advice if experiencing these symptoms.

Submit
197. Which of the following group of condition has lesion of oral mucosa that appear as vesicles during stages of their development:

Explanation

Pemphigus, herpes zoster, and herpes simplex are conditions that can cause lesions on the oral mucosa that appear as vesicles during stages of their development. Pemphigus is an autoimmune disorder that causes blistering of the skin and mucous membranes, including the mouth. Herpes zoster, also known as shingles, is a viral infection that causes a painful rash with fluid-filled blisters. Herpes simplex is a viral infection that can cause cold sores or fever blisters on the lips and mouth. These conditions all have the characteristic of vesicular lesions on the oral mucosa.

Submit
198. The characteristic lesion of the primary stage of acquired syphilis is:

Explanation

The characteristic lesion of the primary stage of acquired syphilis is a chancre. A chancre is a painless ulcer that typically appears at the site of infection, usually the genitals, anus, or mouth. It is a primary symptom of syphilis and is usually accompanied by swollen lymph nodes. Gumma is a late-stage manifestation of syphilis, characterized by soft, tumor-like growths. Mucous patch is a secondary stage symptom, presenting as white, patchy lesions on the mucous membranes. Tabes dorsalis is a late-stage neurological complication of syphilis.

Submit
199. The lesion which are highly infective mimics leukoplakia are seen in:

Explanation

During the secondary stage of acquired syphilis, highly infective lesions that mimic leukoplakia can be observed. This stage occurs a few weeks after the appearance of the primary syphilis sore. The lesions in this stage can appear as white patches on the mucous membranes, including the mouth. These lesions are highly contagious and can easily be transmitted through direct contact. Therefore, the correct answer is the secondary stage of acquired syphilis.

Submit
200. A 57 Y old male comes to dental clinic and presents with is the normal wearing away of tooth structure during mastication (chewing). What would be your most likely diagnosis?:

Explanation

Attrition is the most likely diagnosis for a 57-year-old male who presents with the normal wearing away of tooth structure during mastication (chewing). Attrition refers to the natural process of tooth wear that occurs over time due to normal chewing and grinding of food. It is a common age-related phenomenon and can result in the loss of tooth structure. Abrasion, Bruxism, and Bulimia are not the most likely diagnosis in this case as they involve different causes and symptoms.

Submit
View My Results

Quiz Review Timeline (Updated): Mar 18, 2023 +

Our quizzes are rigorously reviewed, monitored and continuously updated by our expert board to maintain accuracy, relevance, and timeliness.

  • Current Version
  • Mar 18, 2023
    Quiz Edited by
    ProProfs Editorial Team
  • Sep 05, 2019
    Quiz Created by
    Uhsmaster101
Cancel
  • All
    All (200)
  • Unanswered
    Unanswered ()
  • Answered
    Answered ()
Expansion of radicular cyst is due to:
Factors that predispose to the development of denture stomatitis...
A 20 year old female student presents with red patches on her tongue...
The patient with a fissured tongue is advised to :
All sialoliths affect the major salivary glands:
Very common complication of dentigerous cyst is:
Most common etiology for radicular cyst is:
Commonest site for sialolithiasis is:
A biopsy would be of value in the diagnosis or oral lesion of:
Peripheral ossifying fibroma most commonly appears to originate from:
The tumor are soft to touch, movable, generally painless:
The main causes of the lipoma is :
Oral cytological smears are of no value in the diagnosis of:
The term of tumor which is a caused by Human herpesvirus 8 (HHV8), and...
Elderly patients sometimes present with a fungal infection of the...
Many diseases are due to infection with one type of micro-organism or...
Which of the following options is the group of bacteria most likely to...
When body tissues become infected, they exhibit five classic signs of...
Is a mild, but highly contagious viral infection common in young...
Sign and symptom of Hand food moth diseas:
Hand food mouth disease is the infection persons are most contagious...
Signe and symptom of the Halitosis:
Management of the Halitosis:
The following are the factors known to cause Apthouse Stomatitis:
Treatment management of the Apthouse stomatitis:
You are dentist in general practice. A patient attends your practice...
An inflammation or swelling of the tissue lining the sinus, this is...
The maxillary Sinus :
A patient is most likely to experience pain due to infection of the...
Mary 26- year-old woman is worried and embarrassed because of her...
The patient 30 year old came to the dental clinic, Dentist asking her...
Mary 36 year old came to the dental clinic, he Complain of Chronic gum...
The Patient 16 year old, he is the extremely pain the tooth 36 in the...
Which one of the following is least likely indication for antibiotic...
Scarlet fever is a complication of a streptococcal oropharyngeal...
Most common route of entry mycobacterium into the body is via:
Commonest candidial species causing oral candidiasis are:
The patient 30 year old came to the dental clinic, Dentist asking her...
Mary 36 year old came to the dental clinic, he Complain of Chronic gum...
Mary 36 year old came to the dental clinic, he Complain the corner of...
The Patient 16 year old, he is the extremely pain the tooth 36 in the...
Marya, she is 45 year old, she come the dental clinic because she is...
Smoking, alcohol, Drug and accommodation is the:?
The excessive formation of scar tissue is called:
The classical method of examination follows the following sequence: ?
A biopsy would be of value in the diagnosis or oral lesion:
Normal per microliter of the Platelets from:?
The function of the platelets:?Primary hemostasis : platelet plug...
Often called good cholesterol ; because it remove excess cholesterol...
The radiography, pule vitality, Blood test, Biopsy and study model is...
The specific reason for which the patient has presented himself to the...
Cold test is important for dental test, is mean that the:?
We know about inflammation apex, we know about around tooth is the:?
Medical test is the:?
An enamel defect resulting from the incomplete formation of the enamel...
The term of the treatment refer to the?
Head and neck cancer treatment is therapy using the:
Tooth removable, Gum open flap, Bone graft, Oral cancer: take out...
Which of the following statement are true regarding “hemengioma”:
Type lichen Planus is? :
Wickham’s strie are seen in:
The term xerostomia denotes:
All of the following may lead to xerostomia except:
During oral examination of 57 year old man a large white keratotic...
A malnourished child shows ulceration of gingival and punched out...
What are the shape of the Aphthous ulcers?:
Sing and symptoms of the Dry socket is following excepts one:
The lesion size is greater than one centimeter in diameter, They tend...
Common pathological finding of labial mucosa:
The cause of the Benign migratory glossitis is the:
Candidiasis does occur under conditions such as :
White patches (Plaques) on the tongue, inner cheeks, sometimes on the...
The Symptom of Oral thrush candidiasis like:
Oral Mucosa and soft tissue is the:?
These elongated papillae become stained by food and tobacco, producing...
The most effective anti-plaque agent is:
Denture stomatitis is a clinical diagnosis based on the :
All of the following are etiologically associated with aphthous ulcers...
The most common location of dentigerous cysts are except one:
. For severe cases of RAS and ulcerations associated with Behcet’s...
There are factors to lead to the Sialolithiasis:
The most frequent cause of xerostomia is:
For half the patient with erythema multiform the precise cause is:
Most common benign soft –tissue tumor of oral cavity is:
Diagnostic of lipoma depend on the:
A patient presents with asymptomatic, smooth, circumscribed red area...
The “ Ghon complex” is associate with:
There are many different types of bacteria normally present in the...
A patient attends the surgery with bilateral swelling of the parotid...
Hand food mouth disease is cause by :
How many days of the Hand food moth disease, after fever onset,...
The main cause of the halitosis is the:
Sinusitis is inflammation of the sinuses that results in symptoms such...
Which of the following paranasal sinuses open into the middle meatus?:
Burning Mouth Syndrome affects mostly on :
The patient 37 year old came to the dental clinic, he Complain of...
25 year old of the women to come the hospital every year for the...
The patient 37 year old came to the faculty of Dentistry: Complain of...
A 4 year old, child presents with fever, gingival bleeding and...
A 60 Y old mal presents with unilateral facial paralysis, cutaneous...
A 55 Y old edentulous female presents with an erythemathous patch on...
A 35 Y old female presents with clicking in her right...
The characteristic appearance of the tongue in the Scarlet fever has...
Caused by an infection by normal commensal (endogenous) organisms of...
The etiology of the behcet’s syndrome is:
When can nicotine stomatitis regression be seen with cessation of...
A 58 Y old female presents with is the abnormal wearing away of tooth...
The patient 37 year old came to the dental clinic, he Complain of...
The most common viral pathogen cultured from oral infection in...
The patient 23 year old came to the dental clinic, he Complain of...
Dens in dents occurs most commonly in the:
An abnormal disease in the flow of saliva is called:
The most common malignancy found in the oral cavity is:
Head and neck examination to exam the:?
Vesicles or Bullae of the mucosa membrane or skin are seen in all the...
The Peak incidence of gingivitis in children occurs at ages:
Intra oral examination, as a dentist, we should check on:?
Teeth and supporting structures except one:?
A patient present with a small, asymptomatic swelling in anterior...
Prehypertension artery, the systolic and diastolic from:?
This test measures all of the cholesterol in all the lipoprotein...
A patient present with hairy tongue, there is most likely hypertrophy...
The hemostasis: platelet plug and Seconded hemostasis : the...
Site most commonly affected by basal cell carcinoma is:
The chef complain should be recorded in the:?
The clinical presentation of the symptoms of tonsillitis by Virus, The...
The reasons of tooth ache or Pain after dental filling is the:?
The final diagnosis for the dentist decision depend on the ?
Way to success diagnosis depend on clear the:?
The method that can completely cured from illness for getting healthy,...
Types of treatment:?
A medical care that involves the use of medications, either alone or...
The supportive and motivate of patients more effective than using...
A medical care that involves the use of medications, either alone or...
Which one of the following is the most common site of occurrence of...
The cause of oral lichen planus is ?:
Denture sore mouth is:
Aphthous stomatitis are ulcers in the mouth that usually occurs on 3...
Which of the following statement is wrong in relation to denture...
The alveolar osteitis is a complication of having tooth extracted....
48 year old male reported with the chief complaint of ulcer on the...
Common variation of a normal tongue the:
The growth of odontogenic keratocyst mainly takes place in...
Classification of the secondary oral thrush (group II) like:
A 55 Y old female presents with a 6 mouth of history recurrence...
Dry socket is a form of :
Inflammation of the lips is referred to as:
A 70 year old gardener presents with crusting of lower lip for the...
Infectious in oral diseases is a :
A patient presents with soreness and bleeding of gingivae, necrosis of...
Acute pseudomembranous candidiasis on denture-bearing mucosa of the...
A 51 year old Jewish man presents with a 9 months history of mouth...
Desquamative gingivitis cause by:
The term of tumor which is a benign composed of adipose tissue. It’s...
When the body tissues undergo a disease process, certain types of...
In the first months of life, babies often develop resistance to...
Treatment management of the Hand food mouth disease is :
In military tuberculosis, dissemination of microorganism occur via:
What histological form is most common to be found in ameloblastoma ?:
Mary 36 year old came to the dental clinic, he Complain of the...
25 year old of the women to come the hospital every year for the...
The cyst occurring under the tongue, cause by obstruction of a...
The term of the oral examination is the exposed?
The clearance of Examination of Dentistry to : ?
There are three basic rules for every clinical examination except...
Oral cytological smear are of no value in the diagnosis of:
A patient present with a loss of tooth structure due to mechanical...
A patient presented with asymptomatic, smooth, circumscribed red area...
A patient present with an asymptomatic soft, fluctuant swelling of the...
The Teeth and supporting structures and Bony structure is intra oral...
A 25-year old patient with an infection and swelling, which are oval...
The time of onset, Mode of onset, the site, changes in...
The diagnosis is identify the nature and cause of a disease or injury...
A 28-year old patient with present the conditions that cause red,...
The etiology of verucca vulgaris is:
Refers to check-ups (examinations and cleanings) and minor restorative...
Leukoplakia is whitish path or plaque which is:
The main cause of the Halitosis is?:
A 65 year old lady with ill fitting dentures and a very poor oral...
Xerostomia may occur due to any of the following except:
Target lesion are seen in:
Leukoplakia very often precedes the development of a:
A boy shows pearly white thickening of the buccal mucosa and has been...
Size of the minor ulcers is:
Below are the causes of Aphthous stomatitis, except:
Garre’s osteomyelitis is a type of chronic osteomyelitis where there...
A fungal infection of the mouth. It’s not contagious and usually...
Which odontogenic cyst manifests characteristically as a pericoronal...
Diseases and conditions make more susceptible to oral thrush like:
Which of the following type of leukoplakia is more prone to become...
Classification of the primary oral thrush (group I) like:
The name geographic tongue:
The usual oral mucosal reactions to the presence of a sensitizing the...
A 30 year old male presents with a painful ulceration on the gingiva...
Treatment of the halitosis except one:
Site affected of the Burning Mouth Syndrome :
Which of the following group of condition has lesion of oral mucosa...
The characteristic lesion of the primary stage of acquired syphilis...
The lesion which are highly infective mimics leukoplakia are seen in:
A 57 Y old male comes to dental clinic and presents with is the normal...
Alert!

Advertisement